Old Q&A – Musculoskeletal System

A 77-year-old man comes to the office with his wife because of “walking difficulties.” He says that over the past 5 months he has noticed that when he walks or stands for longer than 15 minutes he gets pain and weakness in his thighs. The pain is usually relieved by sitting. Within the past 1-2 years he began to get a “discomfort” in the anterolateral thighs, more in the right lower extremity than the left. He also gets a pain in his right hip, which radiates down to just below his knee. He denies ever having any calf pain. He urinates 2-3 times per night and will lose 1 or 2 drops of urine if he cannot make it to the bathroom in time. His wife has notice that he has a tendency to stand with his knees slightly bent rather than straight legged. He tells you that 16 years ago he began to feel “unsteady on his feet.” He did not fall or experience pain at that time, but he had “pins and needles feelings” in his fingers and feet and “lost the feeling of his feet being attached to the ground.” He saw 2 different doctors at that time, had a myelogram, and was diagnosed with C4-5 damage. He underwent C4-5 intercervical discectomy and osteophyte removal. After the surgery he wore a neck brace for several months and the symptoms remained stable. He noticed that his knee reflexes were stronger after the surgery. 4-5 years ago he began to notice that his right knee would buckle. This resulted in 2-3 falls over a 1-year period. He saw a neurologist who prescribed physical therapy and a cervical collar to be worn at night. He did well and stopped wearing the collar about 1 year ago. Physical examination shows weak, but palpable distal pulses, moderately limited neck range of motion, mild weakness of the deltoids and biceps bilaterally, mild weakness of hamstrings and extensor hallucis longus bilaterally, and a normal sensory exam. He has brisk symmetric deep tendon reflexes and down-going toes bilaterally. Tests of coordination are normal and his gait is normo-based and steady but mildly spastic. Cranial nerve and mental status examinations are unremarkable. The most appropriate next step is to

  A. order an MRI of the brain
  B. order an MRI of the cervical spine
  C. order an MRI of the lumbar spine
  D. perform electromyography and nerve conduction study
  E. send him for an angiogram of the lower extremities
Explanation:

The correct answer is C. The history is typical of that for neurogenic claudication, which is caused by lumbar stenosis. The lower extremity weakness is also typical of bilateral L4/L5 radiculopathies. You would want to do an MRI to confirm the diagnosis and make sure that other pathology such as a spinal tumor was not responsible.

There is no reason to suspect brain pathology in this case (choice A). If there were cranial nerve or mental status abnormalities it should be considered.

His gait disorder is being caused primarily by pain, which is secondary to neurogenic claudication of the lumbar spine. He has known cervical spine problems, which may be contributing by impairing balance. It would also cause weakness of deltoids and biceps by C5 root compression. He may need a cervical spine MRI (choice B) at some point, but it is not primarily causing his walking troubles.

There is no indication for electromyography or nerve conduction (choice D) as the history and exam are sufficient to make this diagnosis. An MRI should be ordered to confirm the diagnosis and rule out other pathology.

An angiogram of the lower extremities might be indicated if you suspected vascular claudication (choice E). More commonly this would involve the calves and distal pulses would not be palpable.

 

A 29-year-old woman comes to the clinic complaining of thumb pain. She tells you that she sustained a fracture of the distal radius from a cycling accident 6 weeks ago. At that time a comminuted dorsally displaced fracture of the distal radius was diagnosed by x-ray of the wrist and a cast was applied. The cast was removed last week and the patient resumed normal activity as a schoolteacher. The pain has gradually gotten worse, and radiates from the base of the thumb through the palm. Physical examination reveals pain of the anatomic “snuff box”. There is normal sensation and motion of the hand. Pain increases slightly with hand or wrist motion, but the grip is strong. There is a normal radial pulse. Repeat wrist x-ray reveals a healed Colles fracture and an incidental note is made of sclerosis of the scaphoid (navicular) bone. The most appropriate next step in the management of this patient is to

  A. order a repeat x-ray of the hand
  B. perform compartment measurements to evaluate for compartment syndrome
  C. perform diagnostic arthrocentesis of the first carpometacarpal joint
  D. perform Tinel and Phalen tests to evaluate for carpal tunnel syndrome
  E. request an urgent orthopedic surgery consult to evaluate for avascular necrosis
Explanation:

The correct answer is E. This patient has avascular necrosis (AVN) of the scaphoid (navicular) bone. This is a recognized complication of hand/wrist trauma. Fractures of the waist of the scaphoid bone compromise blood supply to the bone leading to AVN which produces pain in the anatomic “snuff box” and sclerosis of the scaphoid bone on plain radiograph. Scaphoid fractures are often occult on plain radiographs and, hence, repeat radiographs should be performed if there is “snuff box” pain after hand or wrist trauma. Surgery must be performed to pin the scaphoid together and allow healing of the bone and blood vessels.

The diagnosis of AVN has been established by the history, physical exam, and radiograph results so repeat films are not necessary (choice A). A surgical consult is needed.

Compartment measurements are necessary in the case of a suspected compartment syndrome (choice B). The clinical indicators of compartment syndrome are extremity pain, pulselessness, paraesthesia, and pallor indicating a “perishing” extremity.

Arthrocentesis (choice C) is necessary if infection is suspected. The absence of fever argues against infection. Infected joints may be secondary to direct trauma such as an open fracture, but present within days of the initial injury. Hematogenous infectious seeding of a joint occurs in the setting of sepsis. Arthrocentesis must be done urgently in this setting to allow for antibiotic treatment to avoid joint destruction.

Tinel and Phalen tests (choice D) are used to assess median nerve function in the setting of carpal tunnel syndrome (CTS). CTS is a chronic condition involving the compromise of the median nerve in the carpal tunnel. It is not associated with trauma, but repetitive wrist activity such as typing.

 

 

A 87-year-old man with a long-standing history of smoking, chronic obstructive lung disease, peripheral vascular disease, and multiinfarct dementia is admitted to the hospital under your care for gangrenous involvement of the second and fourth toe of his right foot. An arteriogram shows that revascularization is not feasible. After extensive evaluation the surgeon recommends amputation below the knee. The patient is alert and oriented to person, place, and situation. He has some in short-term memory deficits and higher cognitive functions seem intact. You and the surgeon explain the situation to the patient, and despite his dementia, you both believe that he adequately understands the risks and benefits of the surgery. Another physician and nurse also witness the consent procedure and believe that the patient has full comprehension of the risks and benefits of the surgical procedure. However, the patient’s son, who is the designated proxy in the patient’s power of attorney health care documents, adamantly opposes any surgical intervention. He points out that the patient has indicated in the document that he would not like any aggressive measures to save his life. Despite detailed discussions and several family meetings, the son remains adamant about refraining from any surgical intervention. The most appropriate action is to

  A. arrange for a psychiatry referral to assess patient’s competency
  B. ask the son to find other physicians to care for his father
  C. consult your hospital attorney
  D. discharge the patient form hospital
  E. proceed with a below-the-knee amputation
Explanation:

The correct answer is E. Clearly the most appropriate line of management at this time is the below-the-knee amputation. The patient is competent and has given his informed consent.

A psychiatry consultation (choice A) is not necessary in this case. Although the patient has some dementia with some memory deficits, 3 physicians and a nurse think him capable of making his own decisions. Even though the patient has indicated in his advance directive that he would not like any “aggressive measures,” this does not go into effect until the patient loses complete decision making capacity.

Asking the son to find another physician to care for his father (choice B) is another option, but the son has no legal standing in this situation, since he is disagreeing with his father’s wishes.

Consulting the hospital attorney (choice C) is not necessary as there is no legal issue for the hospital.

Discharging the patient from hospital (choice D) would be unethical..

 

 

A mother brings her 3-month old son to the emergency department stating that “for the past 2 days the infant has been moving the right lower extremity less than the left leg”. She is concerned that the right hip may be dislocated even though she denies any history of trauma. She tells you that the pregnancy was non-complicated and a scheduled cesarean section was completed at 34-weeks gestation secondary to twins. The other twin’s past medical history is significant only for hyperbilirubinemia at birth that resolved without complication after 4 days of phototherapy. Physical examination reveals a well-nourished and developed 3-month-old boy with focal tenderness at the distal right thigh. Funduscopic examination is negative. There are no skin lesions, no regions of ecchymosis, abrasions, or other skin changes. The right thigh and knee have no gross deformity, no signs of trauma. Passive hip range of motion is full and symmetric bilaterally. Pelvis and lower extremity x-rays reveal a 2 mm fracture at the medial metaphysis of the right distal femur. The fracture is minimally displaced. There are no other signs of fracture, dislocation, or other bony pathology. When you explain the results to the mother, she is relieved to know the hip is not dislocated. She states that she needs to get home to her other children and husband. The best course of management at this time is to

  A. inform the mother that the fracture is relatively insignificant and will heal uneventfully. Splint the right leg, give an orthopaedic surgery follow-up appointment for 1 week, and discharge the patient
  B. inform the mother that the fracture is relatively insignificant and will heal uneventfully. The fracture measures only 2mm and is minimally displaced so no splint required. Discharge the patient with a 1-week follow-up appointment in the clinic
  C. complete a skeletal survey and contact the patient’s pediatrician by telephone to inquire if there is any history of child abuse in this family. If the pediatrician relays no significant history and the skeletal survey is negative, then discharge the patient home. Splint the right leg and give a follow-up appointment in the clinic
  D. complete a skeletal survey. If there are no other signs of acute or healed fractures, then splint the right leg and discharge to home
  E. complete a skeletal survey and contact child protective services (CPS). Make arrangements for admission until child protective services completes their evaluation. If necessary, call security and detain the child against the mother’s consent
Explanation:

The correct answer is E. Child abuse can occur in any family. A high index of suspicion may save a child from further abuse, or even death. It should be suspected in any and all scenarios. A medial or lateral metaphyseal fracture of any long bone is classic for child abuse. These fractures are referred to as “metaphyseal corner” fractures. More importantly, this 3-month-old boy probably does not roll over let alone bear any weight in the lower extremities. The child could not have fallen on his own to induce this fracture. Any child who is not walking and has a lower extremity fracture is a victim of child abuse until proven otherwise. There are definitely mechanisms of accidental trauma that could cause this injury. This is a diagnosis of exclusion. Although the physical exam is negative for other injuries, a skeletal survey must be completed. It is not an option, it must be performed. The literature varies in the reported rate of uncovering other injuries. Do not be the physician who is deceived by the history, physical examination, or the family’s social class. Always suspect child abuse, document a complete physical exam, and contact CPS.

To inform the mother that the fracture is relatively insignificant and will heal uneventfully, splint the right leg, give an orthopaedic surgery follow-up appointment for 1 week, and discharge the patient (choice A) is correct except for discharging the patient home. CPS must be contacted in the ER. No growth disturbances are expected.

Inform the mother that the fracture is relatively insignificant and will heal uneventfully since the fracture measures only 2 mm and is minimally displaced and that no splint is required. Discharge the patient with a 1-week follow-up appointment with the pediatrician (choice B) is incorrect. Always splint long bone fractures. They may progress if not protected. At the very least a splint reminds caregivers of the fracture. CPS must be contacted in the ER.

Complete a skeletal survey and contact the patient’s pediatrician by telephone to inquire if there is any history of child abuse in this family. If the pediatrician relays no significant history and the skeletal survey is negative, then discharge the patient to home. Splint the right leg and give a pediatric follow-up appointment (choice C) is incorrect. Lack of a history of child abuse is irrelevant. The patient presents with an injury consistent with abuse. One episode warrants for CPS to be involved. Laws vary by state, but it is safe to assume that all states require all suspected cases of child abuse to be reported to CPS.

This child cannot be discharged before CPS consultation (choice D). Admission should be through any accepting service, orthopaedic surgery, pediatrics, etc. If this patient is discharged, the next visit to the ER may be for a much more significant injury or even death.

 

 

A 51-year-old woman who your partner has been treating for low back pain calls the office complaining of a change in symptoms. She reports that although she has had intermittent low back pain for a number of years, in the past few weeks there has been a profound increase in her pain. She also reports that her left leg often tingles and is numb. In reviewing her records, it is noted that the patient has been seen in the office over the years for back pain and has recently been informed that she can no longer obtain narcotics. The patient works as a daycare manager and often lifts small children. She is married with a 4-year-old daughter. The most appropriate next step is to

  A. ask the patient to come to the office for a neurological evaluation
  B. order an MRI of her lumbar spine
  C. prescribe additional narcotic analgesia
  D. refuse to see the patient as she is malingering
  E. schedule the patient for epidural steroid injections
Explanation:

The correct answer is A. This patient suffers from low back pain. This entity is one the most challenging disorders in all of medicine. It has been estimated that 70% of the population suffers from some sort of low back pain at an annual cost of between $20 and $50 billion. The etiology is often unknown in 85-90% of cases but most cases resolve with support and mild pain relief. The goals of evaluation are primarily to exclude serious underlying visceral or systemic disorders. This patient, despite being seen for back pain in the past, has new neurological symptoms which suggest an underlying etiology that is not simply musculoskeletal. She requires a full physical examination.

In the absence of a full exam, which would point to a specific diagnosis, it is not appropriate to order an MRI of her lumbar spine (choice B) without seeing the patient.

In the presence of new signs or symptoms such as the ones this patient is describing, masking the pain with narcotics (choice C) is incorrect. In addition, narcotics have been shown to be only minimally effective in managing low back pain and are not considered standard management for this condition.

Even though this patient has had a complex past history with the clinic, her new signs and symptoms point towards a specific diagnosis such as spinal stenosis or disk herniation and she requires evaluation. To consider her a malingerer (choice D) without evaluation is inappropriate.

Epidural steroid injections (choice E) is a specific therapy aimed at reducing inflammation in certain conditions such as epidural adhesions, facet degeneration, and refractory low back pain. Their efficacy is uncertain and the treatment is usually attempted only after more serous disease is excluded and the patient has failed physical therapy and oral antiinflammatory agents.

 

 

A 67-year-old man comes to the clinic with a new rash on his upper eyelids that he says he has had for a few months. He also reports to have experienced increasing weakness of his lower extremities, especially when he tries to stand up from a sitting position. He has hypertension, which is well controlled with amlodipine, and hypercholesterolemia controlled with diet and exercise, that he’s no longer able to perform. He denies any recent weight loss. Physical examination reveals a well-nourished male with normal chest and lung exam. A diffuse, ill-defined, violaceous plaque is apparent on each upper eyelid. The oral mucosa is within normal limits. Notable on extremities are firm, violaceous plaques overlying the proximal and distal interphalangeal joints on the dorsum surface. There is no involvement of the space between joints on his hands. The general appearance of his hands resembles a mechanic’s hands. He also has a poikiodermatous patch (hypopigmentation, hyperpigmentation, telangiectatic, and atrophy) involving the V of the neck as well as the upper back. Brief neurologic examination reveals proximal lower extremity weakness. The most appropriate management of this patient is to

  A. determine the antinuclear antibody level
  B. determine anti-Rho and anti-La levels
  C. order a liver function test
  D. order a rheumatoid factor level
  E. search for an internal neoplasm
Explanation:

The correct answer is E. Searching for internal neoplasm is the best work up, because dermatomyositis in adults often appears as a paraneoplastic phenomenon. Progressive weakness is the major clinical manifestation. Difficulty in rising from low chairs, climbing stairs, or holding the arms above the head are common symptoms. The most specific skin manifestation is a maculopapular erythema on the bony prominences, such as the knuckles, elbows, and knees that appears in about 70% of cases. A blotchy erythema, especially on sun-exposed skin, occurs in about a third of patients. More chronic macular erythema and poikilodermatous lesions may also develop on the trunk and proximal parts of the limbs. A distinctive sign is an erythematous blush on the eyelids and around the eyes. It has a lavender shade and is called a heliotrope erythema. Patients over the age of 50 at onset, should be evaluated for the presence of a malignant tumor. Chest x-ray, gastrointestinal evaluation, and careful breast and genitourinary examinations are sufficient unless symptoms point elsewhere.

While antinuclear antibody (choice A) may be positive in dermatomyositis, it is not the best work up nor is it the most diagnostic test.

Anti-Rho and anti-La (choice B) is incorrect, because these are the main markers for Sjogren’s, subacute lupus erythematosus, and neonatal lupus, but not for dermatomyositis.

A liver function test (choice C) is incorrect, because it is neither diagnostic for dermatomyositis nor the best work up.

Checking rheumatoid factor levels(choice D) is incorrect, because dermatomyositis patients do not usually have an elevated rheumatoid factor level.

 

 

A 23-year-old woman comes into the office because of a 1-week history of painful nodules on her legs. She was recently seen in your office and started on oral contraceptives for the first time. About 2 weeks earlier, she had a “cold” which was associated with sore throat and fever, but she recovered without complications. Physical examination shows multiple red, deep-seated, tender, 2-4 cm nodules on the pretibial region of both legs. She says that “the first lesions” have flattened, leaving a purple or blue-green discoloration. No involvement of the ankles or posterior legs is appreciated. The most appropriate management of her condition is to

  A. administer penicillin IM for the presumed upper respiratory infection
  B. admit her to the hospital and begin amphotericin for presumed deep fungal infection
  C. change her to a different oral contraceptive pill
  D. prescribe acetaminophen with codeine
  E. recommend ibuprofen 600mg 4 times per day
Explanation:

The correct answer is E. Ibuprofen or another nonsteroidal antiinflammatory drug is the treatment of choice for erythema nodosum (EN). EN is a common inflammatory panniculitis which occurs in two forms: acute or chronic. Acute EN most commonly occurs in young adult women with bilateral, symmetrical, deep, tender nodules 1-10 cm in diameter. Usually there are up to 10 lesions but many more may be found. The most common location is the pretibial area. The onset is acute, frequently associated with malaise, leg edema, and arthritis or arthralgias (usually of the ankles, knees or wrists). Over a few days, the lesions flatten, leaving a purple or blue green color resembling a deep bruise. It is commonly associated with a streptococcal infection. Other infectious etiologies have been implied including Mycobacteria tuberculosis, Yersinia, Shigella, Salmonella, or fungal infection. Drugs may also induce erythema nodosum including bromides, iodides, and sulfonamides. The most common drug to induce EN is oral contraceptives. Besides treatment with NSAIDs, if lesions do not resolve within weeks of treatment, physicians should investigate for other infectious causes. Discontinuing oral contraceptive pills may also be important.

Penicillin IM (choice A) is incorrect because treating a respiratory infection without positive culture is not the standard of practice. The proper step here would have been performing a throat culture to determine if Streptococcus is the offending organism prior to initiating treatment

Admit to the hospital to start amphotericin (choice B) is incorrect because deep fungal infection does not usually appear in crops in a healthy, immunocompetent patient. This patient is classic for either streptococcal or drug-induced EN.

Changing oral contraceptive (choice C) is incorrect because most oral contraceptives have been associated with development of EN. Rather than changing to a different type of contraceptive, this patient may be better off discontinuing OCPs.

Acetaminophen with codeine (choice D) is incorrect because only NSAIDs or indomethacin are helpful in EN.

 

 

You are seeing an 83-year-old woman for preoperative clearance prior to a total hip replacement. She has a long history of rheumatoid arthritis with pulmonary involvement. Her disease has, however, been well controlled over the last several years on methotrexate. She has no known allergies to any medications. She does not smoke or drink alcohol. She is active and walks a mile, 3 times a week. Her temperature is 37.7C (99.9 F), blood pressure is 110/56 mm Hg, pulse is 89/min, and respirations are 19/min. Her hands show degenerative changes consistent with long standing rheumatoid disease. Her lungs are clear to auscultation and her cardiac rhythm is regular. She has limited neck flexion-extension. A neurologic examination is unremarkable. A chest radiograph is unremarkable. An electrocardiogram shows a sinus rhythm with some non-specific ST and T wave abnormalities. During your discussion, the patient expresses a desire to have a general anesthetic. Based upon the available information, the next most reasonable imaging study to obtain is

  A. a cervical spine radiograph
  B. a chest CT
  C. a head CT
  D. a spine MRI
  E. an echocardiogram
Explanation:

The correct answer is A. Since the patient is insisting on a general anesthetic and will thus require intubation, it is important to ensure that the patient has no cervical disease as a result of her rheumatoid arthritis, especially given her history of limited neck mobility.

A chest CT (choice B) in the setting of a benign lung exam and chest radiograph is unlikely to add useful information in this scenario.

A head CT (choice C) is not indicated in this setting, since there is no reason to suspect any intracranial pathology.

A spine MRI (choice D) is not indicated, given the absence of any historical or physical exam features to suggest spinal cord pathology.

An echocardiogram (choice E) is not indicated given that she is very active and has an unremarkable cardiac exam and electrocardiogram.

 

 

A 28-year-old man comes to the emergency department because of moderate left shoulder pain that is worse with abduction of the shoulder. He plays baseball occasionally with friends and has noticed that the pain worsens when throwing the ball. He denies any history of trauma. Physical examination shows weakness of the shoulder, most pronounced with abduction. A shoulder x-ray reveals no fractures or dislocations. The most appropriate next step in management is to

  A. advise patient that he should rest and ice the shoulder
  B. order an emergent CT scan of the shoulder
  C. order an emergent MRI of the shoulder
  D. order an outpatient MRI of the shoulder
  E. prescribe nonsteroidal antiinflammatory drugs
Explanation:

The correct answer is D. Rotator cuff tears are chronic use injuries most common in throwing athletes. The clinical symptoms are weakness and instability of the shoulder. An x-ray may show a subacromial spurring, a high-riding shoulder, or calcific tendonitis. Diagnosis is made by MRI of the shoulder. This is a non-emergent diagnosis and the imaging can be performed on an outpatient basis. Repair is made arthroscopically.

Advising the patient to rest and ice the shoulder (choice A) will help alleviate the immediate symptoms of pain. It will not, however, treat the underlying condition of a rotator cuff tear. Rest and ice are good for muscle sprains, but will not help torn tendons.

A CT scan of the shoulder (choice B) is not indicated in suspected cases of rotator cuff tears. A CT is good for evaluating the bones of the shoulder, but does not provide the resolution to evaluate the muscles and tendons of the shoulder.

An emergent MRI of the shoulder (choice C) is not indicated. Rotator cuff tears are chronic overuse injuries and do not require emergent diagnosis or repair. The MRI should be ordered on an outpatient basis.

Prescribing nonsteroidal antiinflammatory drugs (choice E) will help alleviate the immediate symptoms of pain. It will not, however, treat the underlying condition of a rotator cuff tear.

 

 

A 35-year-old woman is in the hospital for a flare of nephritis related to systemic lupus erythematosus (SLE). On rounds in the morning, she complains of right hip pain. She states that for the last several weeks, she has had a deep aching in the hip and now it is getting much worse since she was in the hospital. It hurts her both at rest and with motion. She denies any history of trauma, and has not started any new activities. There have been no fevers. Her only outpatient medication is prednisone 10 mg daily, and she takes ibuprofen for pain relief, which has helped minimally. However, now she is on a higher dose of intravenous steroid. Her temperature is 37.2 C (99 F), blood pressure is 132/82 mm Hg, and pulse is 72/min. There is no pain on palpation over the hip but pain is present with range of motion. Laboratory studies show a leukocyte count 8,100mm3 and a hematocrit 34%. A plain x-ray of the pelvis and hip is normal. The next most appropriate step is to

  A. assure her that her pain will resolve spontaneously
  B. continue the patient on oral ibuprofen and follow her symptoms as an outpatient in 1 month
  C. increase the duration of her intravenous steroids
  D. order an MRI of the hip
  E. send her for an arthrogram of the hip
Explanation:

The correct answer is D. In a patient with nontraumatic hip pain, who is on chronic steroid therapy for a disease such as SLE, a major concern is avascular necrosis (AVN) of the bones. AVN passes through several stages, the first of which has a totally normal x-ray. It will progress to patchy sclerosis, then subchondral fracture, cortical collapse, and eventually lead to severe irreversible degenerative change requiring total hip replacement. In the setting of AVN, an MRI will show marked bone marrow edema, even when the x-ray is totally normal and is the only objective test to rule out the disease in this early stage. Treatment early can prevent disease progression. Surgical core decompression is one potential treatment, where a core of bone is taken from the femoral head and replaced with a vascularized fibular graft. Trials are also ongoing to evaluate the effectiveness of packing the core with bone morphogenic proteins to promote healing.

Assuring her that this will resolve spontaneously (choice A) or following up as an outpatient in 1 month (choice B) are not good options. The longer this disease goes unrecognized, the greater chances are that it will progress to the more advanced stages.

Systemic corticosteroids are actually the underlying cause of the AVN rather than the SLE itself. In other diseases, such as sickle cell, the disease itself, actually results in the AVN. In any case, increasing the dose of prednisone (choice C) would actually exacerbate the problem.

An arthrogram of the hip (choice E) would not be useful in this diagnosis, as it will evaluate the joint space, but will not add any information about what is going on within the bone marrow.

 

 

A 67-year-old man comes to the clinic with a lesion on his lower lip. He tells you that this lesion started out as a “pimple” and has been increasing in size over the past 8 months. He has been smoking 2 packs of cigarettes a day for the past 40 years. Physical examination shows an ulcerated 0.9 cm nodule on the lower lip. There are multiple small, firm, non-mobile lymph nodes in the cervical region. He appears to have sun damage on his face. At this time the most correct statement about his condition is:

  A. His history of sun exposure is more important than smoking in the development of this lesion
  B. This is most likely basal cell carcinoma of the lip requiring lip wedge resection
  C. This is most likely leukoplakia of the lip
  D. This is most likely squamous cell carcinoma and a referral for mass and lymph node resection followed by hematology/oncology evaluation is indicated
  E. Squamous cell carcinoma of the lip has the same prognosis as squamous cell carcinoma of the face
Explanation:

The correct answer is D. This is most likely squamous cell carcinoma (SCC) and a referral for mass and lymph node resection followed by hematology/oncology evaluation is indicated. SCC of the lip is as aggressive as SCC of oral mucosa and is commonly associated with smoking, smokeless tobacco, betel nut chewing, and alcohol consumption. It occurs primarily in older men. The most frequent sites are the lower lip, tongue, soft palate, and the floor of the mouth. Unfortunately, the survival rate is only 30% because squamous cell carcinoma is often discovered late, often it has metastasized to the cervical lymph nodes. The best approach for this patient is referral to otolaryngology for mass and neck resection and then followed by adjuvant radiation or chemotherapy as determined by an oncologist.

History of sun exposure is more important than smoking (choice A) is incorrect. Oral SCC can be associated with actinic damage, but tobacco and alcohol are the most common risk factors for head and neck cancers.

Basal cell carcinoma (choice B) is unlikely because these lesions tend to have soft, friable and pearly appearances with telangiectasias.

Leukoplakia of the lip (choice C) is incorrect as this term refers to whitish plaques or patches in mucous membranes, which may or may not show cellular atypia.

Squamous cell carcinoma of the lip has the same prognosis as ones of the face (choice E) is incorrect. The prognosis of SCC on face (i.e. forehead) is excellent when caught early and complete resection has a greater than 85% cure rate, whereas, oral SCC has poor prognosis in general and grows much more rapidly.

 

 

A 64-year-old man comes to the clinic because of a “spot” on the side of his face that has been there for about 8 months. He says that he is an executive at a local company and is retiring at the end of the year. The company has hired a portrait artist to paint his picture that will hang in the boardroom for many years to come, and so he realized that this is a good time to “have this thing taken off.” He thinks that the lesion has not grown since he noticed it, but he has not paid it much attention. He plays tennis every weekend at his country club and then lies in the sun with his wife. This is the first time you have seen this patient, but he tells you that he has been very healthy and has only suffered through “a couple of bouts of kidney stones” over the years. Physical examination shows a 2.3-cm waxy, verrucous, dark brown papule with a “stuck-on” appearance.The most likely diagnosis is

  A. actinic keratosis
  B. basal cell carcinoma
  C. dermatofibroma
  D. melanoma
  E. psoriasis
  F. seborrheic keratosis
  G. squamous cell carcinoma
Explanation:

The correct answer is F. This patient most likely has seborrheic keratosis. This lesion is characterized by light brown to black papules or plaques with an adherent waxy, greasy scale. The “stuck-on” appearance is very characteristic. It is most often found on the face and trunk.

Actinic keratosis (choice A) is characterized by flesh-colored or red to brown macules or papules with a rough scale. It is most common on sun-exposed skin.

Basal cell carcinoma (choice B) is characterized by a papule with a central ulceration and a pearly, raised border. It is most common on sun-damaged skin.

Dermatofibroma (choice C) is characterized by a firm, red to brown nodule with a dimpling of the skin. It can occur anywhere.

Melanoma (choice D) is characterized by an asymmetric, pigmented lesion with an irregular border. Any change in an existing skin lesion should be evaluated for melanoma.

Psoriasis (choice E) is characterized by erythematous plaques and papules covered with silvery scales. The most common sites are elbows, knees, scalp, and back. Pitting of the nails may be present.

Squamous cell carcinoma (choice G) is characterized by a hyperkeratotic, ulcerated, or crusty lesion. The face, especially the lower lip, is the typical site of involvement.

 

 

A 15-year-old boy who has been HIV-positive since birth is admitted to the hospital because of severe lower back pain. He denies a history of trauma and “has no idea how this started”. His temperature is 37 C (98.6 F), blood pressure is 140/70 mm Hg, pulse is 100/min, and respirations are 19/min. Physical examination shows point tenderness of L4 and L5 posteriorly. A neurologic examination is otherwise unremarkable and there is a normal gait. Laboratory studies show a leukocyte count of 15,000/mm3. An MRI of the lumbar spine reveals inflammation of the L4 and L5 vertebral bodies and the L4-5 intervertebral disc. There is a focal fluid collection in the L4-5 disc space. There is no cord compromise or abscess. The next step, after initiating broad-spectrum antibiotic therapy is to

  A. aspirate the L4-5 disc space under fluoroscopic guidance
  B. prepare him for an exploratory laminectomy and decompression
  C. order lower extremity nerve conduction studies
  D. send him for a non-contrast head CT
  E. send STAT blood cultures
Explanation:

The correct answer is A. This patient has symptoms and imaging findings consistent with L4-5 discitis. Given the immunocompromised status of the patient, a wide variety of organisms are likely pathogens. In this case, hematogenous spread to the disc space is likely. It is essential to begin immediate broad-spectrum antibiotic therapy and obtain tissue in an attempt to isolate an organism. Systemic blood cultures will certainly be drawn, but they are low yield. Tissue must be obtained under imaging guidance to isolate an organism and narrow antibiotic coverage.

Exploratory laminectomy and decompression (choice B) would be necessary only in the setting of an abscess that compromises the spinal cord. A repeat MRI should be performed if symptoms of cord compression develop.

Lower extremity nerve conduction studies (choice C) are used in the setting of suspected peripheral neuropathy. The physical exam is normal in this case, so there is no indication of spinal cord compression at this time.

Non-contrast head CT (choice D) is necessary prior to lumbar puncture to exclude a cause of increased intracranial pressure that could lead to herniation during or after a lumbar puncture. A lumbar puncture is not necessary here given the lack of neurologic and MRI findings of cord compromise. Blood cultures (choice E) must be drawn, but they are unlikely to be positive given the lack of fever. Obtaining tissue for staining and culture, at the infected site, is paramount.

 

 

A 55-year-old man comes to the emergency department because of numerous pustules associated with fever, fatigue, and arthralgias over the past 2 days. He has a history of “eczema” and was seen by a dermatologist 10 days prior due to a “flare-up of his eczema.” He was given tapering doses of oral corticosteroids, which he completed 3 days ago. On further questioning you learn that his father and brother both have severe psoriasis. His temperature is 38.8 C (101.8 F). He has dry oral mucosa and generalized erythema with hundreds of small, pinpoint pustules involving the entire skin surface, some coalescing into large pools of pustule pockets. There are multiple “oil drops” of pitting in the fingernail beds. At this time the most correct statement is that

  A. the appropriate management would be to culture the pustules and admit the patient for intravenous antibiotic therapy
  B. he has pustular psoriasis and the offending culprit is the systemic corticosteroids
  C. he has scalded skin syndrome and oral antibiotic therapy is indicated.
  D. he has toxic epidermal necrolysis and you need to start intravenous immunoglobulin treatment
  E. this is a pustular drug eruption; the patient is allergic to prednisone
Explanation:

The correct answer is B. Pustular psoriasis is characterized by acute onset of sheets of sterile pustules on a background of erythema. The pustules often become confluent, forming lakes of pus. The skin is painful and tender. The pustules are accompanied by fever, malaise, arthralgia, and diarrhea. Pustular psoriasis in known to be triggered by systemic corticosteroids as a rebound phenomenon in a person with known psoriasis. In this patient, his previously treated “eczema” may have been a misdiagnosed psoriatic plaque, given his strong family history of psoriasis. Treatment includes supportive care, intravenous hydration, and systemic cyclosporine.

To culture the pustules and admit for intravenous antibiotic (choice A) is incorrect because as mentioned above, pustular psoriasis is composed of sterile pustules and an antibiotic will not alleviate the flare.

Scalded skin syndrome (choice C) is incorrect because this is a disorder generally seen in adults with immunosuppression or renal impairment. Clinical manifestation includes malaise, fever, irritability, a generalized macular erythema, and a fine, stippled, sandpaper or nutmeg-like appearance that quickly progresses to a tender scarlatiniform phase over 1 to 2 days. From the intertriginous and periorificial areas and trunk, the erythema and tenderness spread over the entire body, usually sparing the hairy parts. This is followed by an exfoliative phase, heralded by exudation and crusting around the mouth and sometimes the orbits.

Toxic epidermal necrolysis (choice D) is incorrect because this is a disorder with widespread macular erythema and stomatitis with exfoliation of the skin. As the erythema becomes more universal, the skin begins to develop areas of separation between epidermis and dermis. The pathogenesis of toxic epidermal necrolysis involves a drug-induced necrosis of the basal cell layer of the epidermis, with the production of subepidermal separation rather than a superficial split as seen in scalded skin syndrome.

Pustular drug eruption (choice E) is incorrect because prednisone can cause acneiform eruption at follicles but does not usually cause pustules that coalesce into lakes. In addition, the strong family history of psoriasis in this patient should lead you to a diagnosis of pustular psoriasis.

 

 

A 32-year-old gardener comes to the office because of a lesion on his nose that has been increasing in size over the past few weeks. He says that he rarely sees a doctor and is reluctant to discuss his medical history. He works outside for 9 to 10 hours a day, smokes two packs of cigarettes a day, and drinks approximately a case of beer a night. He states that he sometimes “shoots up” heroin with a few guys that “hang out on a stoop around the block.” Physical examination shows a 1.5-cm, purple nodule on the tip of his nose, a 0.6-cm, reddish-purple raised macule on the tragus of his left ear, a 1-cm, purplish-blue area of discoloration on the roof of his mouth, and a 4-cm, confluent lesion made up of purplish-brown plaques, patches, nodules, and tumors on the anterior surface of his right leg. The results of the punch biopsy, which return in one week, show spindle cells, endothelial cells, and the extravasation of red blood cells. The factor in this patient’s history most closely correlated with these lesions is

  A. alcohol intake
  B. cigarette smoking
  C. intravenous drug use
  D. pesticide exposure
  E. sun-exposure
Explanation:

The correct answer is C. This patient most likely has Kaposi’s sarcoma, which is one of the most common malignancies affecting HIV-infected individuals. It is a vascular tumor characterized by purplish lesions (macules, plaques, nodules, and tumors) that occur most commonly on the face, oral mucosa, lymph nodes, lungs, and gastrointestinal tract. Biopsies of the lesions show spindle cells, endothelial cells, and the extravasation of red blood cells. Management of Kaposi’s sarcoma is dependent on the size, location, and the severity of other diseases, and includes observation, surgery, radiation, biological therapy, and chemotherapy. This patient was most likely infected with HIV from intravenous drug use.

Alcohol (choice A) and cigarette smoking (choice B) are not associated with Kaposi’s sarcoma.

Pesticide exposure (choice D) may be associated with contact dermatitis, which is a cutaneous response to an irritant. The typical characteristics are pruritus, erythema, edema, and vesicles. Kaposi’s sarcoma is not associated with pesticide exposure.

Sun-exposure (choice E) is associated with basal-cell carcinoma, squamous cell carcinoma, and malignant melanoma. Basal-cell carcinoma typically occurs on the face and is characterized by a papule with central ulceration and a “pearly” border. Histological evaluation shows nests and clusters of deeply basophilic epithelial cells with “palisading” nuclei on the periphery. Squamous-cell carcinoma typically occurs on the face, especially the ears and lower lip, and is characterized by an indurated and hyperkeratotic plaque with crusting and ulceration. Histological evaluation shows sheets and nests of neoplastic epidermal cells. Malignant melanoma, which has many clinical variants, is typically associated with sun exposure in childhood. It can occur almost anywhere on the skin or on the body and can be melanotic or amelanotic. The typical lesions occur on the trunk, and extremities, are irregularly pigmented, and have asymmetric borders. Microscopic evaluation shows neoplastic melanocytes. Kaposi’s sarcoma is not associated with any of these malignancies.

 

 

A 69-year-old woman with hypertension and mild anxiety disorder comes to the clinic because of a 10-day history of lower lumbar pain. She tells you that she is unsure of how it started and cannot give you a more detailed history. She tells you that she had a hysterectomy and oophorectomy for unknown reasons 40 years ago. She takes atenolol, a multivitamin, calcium supplements, and alprazolam. Physical examination is unremarkable. Plain radiographs of the lumbar spine demonstrate multiple age indeterminate lumbar compression fractures and hyperlucent vertebra. A pelvic ultrasound reveals no uterus or ovaries and a normal aorta. The most appropriate next step in the clinic today is to

  A. recommend calcitonin and estrogen replacement therapy
  B. recommend calcitonin, estrogen replacement therapy, and ibuprofen
  C. recommend estrogen replacement therapy and ibuprofen
  D. recommend estrogen replacement therapy only
  E. send her for x-ray bone densitometry
Explanation:

The correct answer is B. This patient has severe osteoporosis with compression fractures. Her disease is severe because her ovaries were removed at a young age. Standard treatment includes estrogen replacement and calcium supplementation along with moderate exercise. In women who are at least 5 years beyond menopause, calcitonin slows bone loss, increases spinal bone density, and, according to anecdotal reports, relieves the pain associated with bone fractures. Calcitonin reduces the risk of spinal fractures. In addition, the pain of compression fractures, which can be severe, must be managed with an analgesic such as ibuprofen.

Calcitonin therapy with estrogen replacement (choice A) along with calcium supplementation is aggressive treatment for osteoporosis. But this patient has known compression fractures and pain, so analgesia is indicated.

Estrogen replacement with ibuprofen (choice C) along with calcium supplementation fails to utilize the beneficial effects of calcitonin in this patient population (see above).

Estrogen replacement (choice D) is only a part of a complete treatment program in an osteoporotic patient with compression fractures. Calcitonin to help increase bone density and analgesia for the compression fractures are necessary.

X-ray bone densitometry (choice E) measures the density of bone compared to normal standards. The presence of compression fractures means that there is already severe osteoporosis. That said, x-ray bone densitometry would be useful to follow the effects of treatment. It would not, however be the first priority at this clinic visit.

 

 

A 19-year-old man is brought to the office by his mother who is concerned about her son’s posture. She states that despite verbal encouragement, physical therapy, and sports participation, her son slouches forward. She inquires, “Why is his back so round?” The mother further relates that previously she was informed his back was normal and he would “grow out of it”. She feels that the roundness has actually been progressing. The patient denies any recent weight loss or gain or change in appetite. He denies any fever, chills, night sweats, or other constitutional symptoms. He denies any bowel or bladder dysfunction. There is no history of weakness, extremity paresthesia, or gait abnormalities. He notes very occasional back pain unrelated to any particular inciting activity. The pain is non-radiating, does not require analgesic, and is relieved with rest. There is no history of trauma. He is also bothered by the appearance of his back stating, “In the summer or while swimming at gym class I usually wear a T-shirt”. The past medical and surgical history is significant only for asthma and an appendectomy in childhood. Family history is non-contributory. He denies alcohol or tobacco use. Physical examination reveals a healthy appearing male of average height and slight obesity. Inspection of the spine region reveals a marked thoracic kyphosis and increased lumbar lordosis. No focal spinal tenderness or soft tissue changes. Gait is full and symmetric. Neurovascular examination is normal. X-rays were obtained on 3-foot cassettes of the thoracolumbosacral spine in AP and lateral projections. The AP projection reveals an 8-degree curve in the coronal plane from thoracic vertebral body number 6 to number 11. The convexity of this curve is to the right. A left convex curve of 7 degrees is noted from the thoracic body number 12 to lumbar body number 4. The lateral spinal x-ray reveals a thoracic kyphosis of 60 degrees. The apex of the kyphosis is at thoracic body number 9. The lateral radiograph also reveals the vertebral body height is 4 mm greater posteriorly than the anterior height at thoracic vertebrae levels 8, 9, and 10. At this time the most correct statement about his condition is:

  A. Inform the family of the benign nature of this condition and that it is essentially a cosmetic deformity. Emphasize the fact that the patient is having minimal back pain. The only corrective modality is surgery, but the benefits do not outweigh the risks. Schedule the patient for clinical and radiographic follow-up appointments every 6 months. Recommend continuation of physical therapy types of exercises at home as previously performed at physical therapy.
  B. Inform the family that this is most likely a benign cosmetic deformity. However, laboratory studies, including a CBC, ESR, metabolic panel including calcium and phosphorus levels should be completed. A bone density study should also be performed to rule out osteoporosis as the cause for the vertebral height differences. These studies will help rule out infection or a metabolic process as the etiology of the thoracic kyphosis.
  C. This is a benign condition requiring conservative management. Discuss with the family the importance of a well-structured physical activity program. Write a new prescription for 8 weeks of physical therapy. The new therapy script should emphasize strengthening and range of motion exercises for the back and abdominal regions and a home exercise program.
  D. This is a benign condition requiring conservative management. Discuss with the family the importance of a well-structured physical activity program. Write a new prescription for 8 weeks of physical therapy. The new therapy script should emphasize strengthening and range of motion exercises for the back and abdominal regions and a home exercise program. A hyperextension brace should be prescribed and worn 23 hours a day. If the thoracic kyphosis does not improve in 6 months, then an orthopaedic surgery referral is indicated.
  E. This is a benign condition requiring conservative management. Discuss with the family the importance of a well-structured physical activity program. Write a new prescription for 8 weeks of physical therapy. The new therapy script should emphasize strengthening and range of motion exercises for the back and abdominal regions and a home exercise program. In addition, a hyperextension brace should be prescribed and worn 23 hours a day.
Explanation:

The correct answer is A. This patient has Scheuermann’s kyphosis. As described by the Danish radiologist Holger Scheuermann, it is a round back associated with wedged thoracic vertebrae. The etiology remains unclear. Theories have included avascular necrosis of the vertebral bodies, abnormal proteoglycans, and osteoporosis of the vertebrae. Hamstring tightness and hip flexor tightness have been postulated. It is more common in males. Patients may complain of occasional aching back pain, but the most common reason for presentation is the appearance. The parents, and sometimes the patient, are concerned about the cosmetics of the condition. A complaint of back pain may be the patient’s way of complaining about the social pain of the deformity. Increased thoracic kyphosis results in a compensatory increased lumbar lordosis. Normal thoracic kyphosis is less than 45 degrees. A 60-degree kyphosis is moderately severe. Radiographs of patients with Scheuermann’s kyphosis reveal thoracic kyphosis greater than 45 degrees, with 5 degrees or more of anterior wedging of 3 adjacent vertebrae. Other x-ray findings include Schmorl’s nodes, end plate irregularities, disc narrowing, scoliosis, and spondylolysis. Physical exam is otherwise unremarkable, except for hyperkyphosis and a sharply angulated bend at the thoracic or thoracic-lumbar junction. Adolescents with a fixed or progressive kyphosis require postural awareness and an exercise program. Bracing is an option for the skeletally immature and is more successful for curves less than 75 degrees. Teenagers often reject braces. Painful, severe curves may be corrected surgically. Surgery typically requires an anterior spinal release followed by a posterior spinal fusion and instrumentation. The natural history is that most patients will have a worsening of the deformity until skeletal maturity. Kyphosis less than 60 degrees at skeletal maturity will not predictably have symptoms in adult life. The differential diagnosis includes postural round back. This entity has thoracic kyphosis, but there is no vertebra wedging.

This patient does not have an infection, tumor, metabolic disease, or other pathology. Further tests (choice B) are expensive and unnecessary.

This is a benign condition in a skeletally mature patient. Sending the patient back to physical therapy (choice C) is unnecessary. Home activity is warranted for all patients, but will not reverse the deformity of this patient.

Hyperextension braces (choice D) are helpful in the skeletally immature, compliant patient. This patient is skeletally mature, so bracing will be ineffective.

This is a moderate, relatively non-painful kyphosis. The coronal curves are not significant. By definition these curves are not scoliosis, since they are less than 10 degrees. Observation is the management with the continuation of a home exercise program. Spine surgery (choice E) is fraught with potential adverse outcomes, the least of which can be no improvement of the deformity to paralysis, death, etc.

 

 

You are called to see a 45-year-old nursing home resident, who has been there since a motor vehicle accident that left him paralyzed from the neck down 2 months ago. He denies any active medical problems prior to his car accident 2 months ago. In the past month, he has noticed a rash on his back that is occasionally pruritic. He denies any systemic manifestations associated with the rash. He is confined to his bed and the nursing staff turns him to his side once per day by propping him back with multiple pillows. He has notable atrophy of all the extremities. Cutaneous examination reveals numerous non-folliculocentric inflammatory papules distributed over his posterior trunk. There is no involvement of the anterior trunk, extremities, face, or oral mucosa. The most appropriate management of this patient’s condition is to

  A. prescribe oral minocycline
  B. prescribe a topical corticosteroid
  C. prescribe a topical tretinoin
  D. recommend a topical benzoyl peroxide
  E. tell the nurses to rotate the patient on his sides more frequently and keep his room at a cooler temperature
Explanation:

The correct answer is E. Rotating the patient and keeping the room at a cooler temperature is correct because miliaria, also known as heat rash, is a common phenomenon in individuals during prolonged bedrest. It usually involves the posterior trunk of a bedridden patient. The eccrine sweat duct occlusion is the initial event. The duct ruptures, leaks sweat into the surrounding tissues, and induces an inflammatory response. The papular and vesicular lesions resemble folliculitis with one major distinguishing feature. They are not follicular and therefore, do not have a penetrating hair shaft. The best treatment for miliaria is to keep the areas involved cool and avoid occlusive clothing.

Oral minocycline (choice A), topical tretinoin (choice C), and topical benzoyl peroxide (choice D), are incorrect because these are treatments for folliculitis or acneiform eruptions. Since the inflammatory papules on this patient are described as non-folliculocentric, folliculitis is not likely the diagnosis.

A topical corticosteroid (choice B) is incorrect because use of corticosteroids topically may result in miliaria as an adverse reaction.

 

 

A 73-year-old obese man comes to the emergency department complaining of a several day history of a very sore mouth and a 1-day history of “blisters” in his groin, underarms, and back. He was feeling “fine” before this, but has been generally feeling “worse and worse” since this began. He tells you he has a history of “reflux” and gout and had an appendectomy in his youth. He tells you that he takes allopurinol and omeprazole and that the dose of allopurinol was increased about 5 weeks ago. His temperature is 37.0 C (98.6 F). The oral cavity has multiple erosions on the hard palate, buccal mucosa, and gingival surfaces and his breath is foul smelling. The glans penis has similar erosions. His axillae, inguinal area, and back display a mixture of erosions and flaccid bullae filled with a yellowish fluid. Lateral pressure on the skin surrounding the bullae cause sloughing of the epidermis. Biopsy of a representative bulla is obtained and sent for fresh frozen section, revealing a prominent intraepidermal bulla with prominent acantholysis. At this time, the most important therapeutic measure is to

  A. discontinue allopurinol
  B. initiate high-dose corticosteroids
  C. initiate methotrexate therapy
  D. initiate metronidazole therapy
  E. initiate nafcillin therapy
Explanation:

The correct answer is B. This case represents a classic history, physical exam, and histology of pemphigus vulgaris, a bullous disorder characterized by lesions starting at the mouth and mucous membranes and then appearing in the intertriginous areas such as the axilla or groin. The bullae easily rupture leaving erosions; this is especially prominent in the oral cavity where the initial bullae are almost never observed and instead only erosions are seen. The bullae are flaccid (as opposed to bullous pemphigoid, the other classic bullous disorder where they are tense bullae) and are filled with a clear straw colored fluid (representing serum). The surrounding skin demonstrated the Nikolsky sign—a sloughing of the skin with lateral pressure—similar to toxic epidermal necrolysis. There is an autoantibody in these patients that is directed against desmosomes (cellular connections which hold the cells in the epidermis together) accounting for the formation of bullae and the histological finding of acantholysis (separation of the cells of the epidermis from each other). Please note that the history and physical exam alone is sufficient to obtain the diagnosis in this vignette—knowledge of the histology, while confirming the diagnosis, is not necessary to answer the question. Until the discovery of corticosteroids, this disease was universally fatal. High-dose corticosteroids are the cornerstone of therapy and should be instituted as soon as possible.

Discontinuing the allopurinol (choice A) would be appropriate if the patient had toxic epidermal necrolysis (TEN). Like pemphigus vulgaris, TEN shows prominent oral involvement and a positive Nikolsky sign, but bullae are not a prominent feature of the disease. Both typically severely affect the oral cavity and mucosal surfaces, but TEN tend to favor the extremities and trunk whereas pemphigus favors the intertriginous areas. While rarely clinical uncertainty exists between the two, in general the two are not confused. A biopsy of TEN would show a subepidermal bullae with prominent epidermal necrosis. Anticonvulsants, allopurinol, and sulfa containing drugs are the most common cause of TEN.

After pemphigus vulgaris is brought under control with steroids, starting methotrexate (choice C) or other steroid-sparing agents such as cyclophosphamide, azathioprine, or cyclosporine is often initiated. These are not appropriate, however, as first-line agents in the treatment of pemphigus vulgaris.

Initiating metronidazole (choice D) and nafcillin (choice E) would not be useful in controlling pemphigus vulgaris, but if at a later time the patient were to develop a bacterial infection they might be essential.

 

 

A 19-year-old college student was seen in the emergency department 4 days ago for a painful lump on her buttocks. She does not remember what “diagnosis” the emergency department physician gave her, but she was instructed to apply warm compresses to the lump, keep the area covered with a dry gauze, and take an antibiotic (the name of which she does not remember). She now comes to you stating that the gauze is getting dirty with foul-smelling drainage. She is ruining her clothing and has become quite concerned. She denies any fevers, chills, weight loss, trauma to the area, or similar prior episodes. There is no contributory past medical, surgical, familial, or social history. Her temperature is 37.0 C (98.6 F). Her abdomen is soft, nontender, without organomegaly or palpable masses. There is no inguinal adenopathy. Pelvic examination reveals no vaginal discharge, cervical motion tenderness, or adnexal masses. There is a 2 cm mass between the superior area of her gluteal folds. The mass is red, warm, fluctuant, and very tender. You notice an area of drainage from the mass. Rectal examination shows good sphincter tone without masses, tenderness, or fluctuance. Stool is guaiac-negative. The most likely diagnosis is

  A. an enterocutaneous fistula
  B. genital warts
  C. an infected Bartholin’s cyst
  D. an infected pilonidal cyst
  E. metastatic rectal cancer
Explanation:

The correct answer is D. This woman is suffering from an infected pilonidal cyst. This cyst, sometimes called a pilonidal sinus, is caused by infection of the hair follicle in the sacrococcygeal area. It may present as an acute abscess at the sacrococcygeal area that may rupture spontaneously. The most common presentation is a painful and fluctuant mass. Initially only cellulites may be present. However, in the chronic stage, the diagnosis is confirmed by the presence of a sinus opening in the intergluteal fold, about 5 cm above the anus. Examination may reveal a pit or pits in the midline that represent infected hair follicles. Treatment of the acute infection includes incision and drainage of the sinus. All hair must be removed. Long-term treatment may require extensive excision of all midline pits or sinuses and removal of all hair and debris.

An enterocutaneous fistula (choice A) is an abnormally draining sinus occurring between the colon and the skin. The foul smelling drainage from this woman’s abscess is pus, not stool. Patients with enterocutaneous fistulas have compromised immune function and usually have undergone some form of bowel surgery. The fistula is usually on the anterior abdominal wall.

Genital warts (choice B) may be painful depending on the etiology. They present on the labia, not the gluteus.

An infected Bartholin’s cyst (choice C) occurs when the Bartholin’s gland has a duct (which drains into the distal aspect of the vagina), that becomes obstructed. This obstruction causes the mucoid secretion to collect and causes dilatation of the cyst. This collection may get infected. Therapy requires incision and drainage of the gland or possibly marsupialization of the gland.

This patient is quite young to have rectal cancer (choice E), let alone metastatic rectal cancer. The combination of no masses on rectal exam, guaiac-negative stool, no family history, and the location of this mass makes this answer highly unlikely.

 

 

A 57-year-old woman comes to the clinic with hoarseness, shortness of breath, cough, and bilateral otalgia for the past 2 days. She tells you that she has had similar symptoms in the past several days, but with much less severity. Her past medical history is significant for moderate arthritis of the knees, ankles, and wrists for the past 30 years. An extensive evaluation for rheumatoid arthritis has been negative. Vital signs are: temperature 37 C (98.6 F), blood pressure 120/90 mm Hg, pulse 82/min, and respirations 9/min. On physical examination, the external ears are tender to touch with the exception of the lobule of the ear. There is no conductive or neural hearing loss. There is a saddle nose deformity. The lungs have minimal bibasilar rhonchi. The abdominal examination reveals a normal size spleen and liver. The ankle and knee joints are tender but not erythematous. Chest x-ray demonstrates focal tracheal narrowing. The most appropriate management at this time is

  A. an arthrocentesis of the knee joint
  B. to order knee and ankle x-rays
  C. to prescribe corticosteroids
  D. to prescribe nonsteroidal antiinflammatory drugs
  E. to schedule a bronchoscopy
Explanation:

The correct answer is C. These symptoms are consistent with relapsing polychondritis (RP). Relapsing polychondritis is a recurrent inflammatory condition of the cartilage most prominent in the joints, nose, ears, costal cartilage, larynx, and airways. Aortic aneurysm may also be a complication. The characteristic clinical feature differentiating RP from other rheumatological conditions is inflammation confined to the cartilage organs like the ears and nose. Tracheal narrowing is a characteristic radiographic finding. Corticosteroids are the main form of treatment of RP. High-dose oral prednisone is used to treat respiratory tract involvement, and intravenous pulse steroids may be useful for acute airway obstruction.

An arthrocentesis (choice A) is not useful in the management or diagnosis of relapsing polychondritis (RP). Synovial fluid samples from patients with RP will show a nonspecific noninflammatory transudate.

Knee and ankle x-rays (choice B) will not contribute to the diagnosis of relapsing polychondritis (RP). RP manifests as a nondeforming, nonerosive arthritis and x-rays of the painful joints are often normal.

Nonsteroidal, antiinflammatory drugs (choice D) have been shown to control mild episodes of inflammation. However, corticosteroids are the mainstay of treatment for relapsing polychondritis.

Bronchoscopy (choice E) can exacerbate laryngotracheal inflammation and be a cause of iatrogenic airway obstruction. Hoarseness, signifying laryngotracheal inflammation, is likely to develop during the course of relapsing polychondritis. Other symptoms of laryngotracheal involvement include cough, dyspnea, wheezing, and choking. Involvement of the airway may be localized or diffuse. The larynx and upper trachea are affected most frequently. Persistent inflammation can destroy the cartilaginous rings and create luminal collapse. This patient already has tracheal narrowing and a bronchoscopy may worsen her airway disease.

 

 

A frustrated young mother brings in her 2-year-old son with spina bifida for a routine check. She reports her son’s rash, that was present at the last visit, has been worsening. At the last visit you went through the atopic dermatitis handout with her and recommended soft, gentle soap and laundry detergent in addition to lukewarm, short baths, instead of steaming hot, long baths. She tells you that she followed the directions carefully, and applied emollients on her son’s skin after baths everyday. Yet, the rash continues to worsen. On examination, you note erythematous, lichenified plaques on his groin areas, upper thighs, and abdominal surface. There is no superficial scaling and no central clearing of these lesions. There is redness and maceration of the perianal area. On further questioning, she reveals the perianal rash is from stool incontinence, and even though she tries to clean him frequently, the area remains red. She proudly adds that she is very clean and wears latex gloves every time she cleans her son. The most appropriate management for the rash on the groin and abdomen is

  A. oral antibiotic therapy
  B. oral antifungal agents
  C. topical antifungal therapy and the use of non-latex gloves
  D. topical antibiotic therapy and the use of non-latex gloves
  E. topical steroid cream to the rash twice a day for 2 weeks and the use of non-latex gloves
Explanation:

The correct answer is E. Using of non-latex gloves is correct because this child has developed a latex allergy. 3-10% of spina bifida patients eventually develop sensitization to latex, hypothesized to be secondary to the increased exposure to surgical procedures in their early life. The history of using latex gloves to clean him regularly for bowel movements as well as erythematous plaques in groin and abdominal areas should raise the suspicion for latex allergy. Half of these patients will have concomitant fruit allergies to banana, avocado, kiwi, chestnut, and passion fruit. The best treatment is to avoid latex material and treat flares with strong topical steroids.

Oral antibiotics (choice A) and topical antibiotics (choice D) are incorrect, because while these two treatment options may decrease the chance of superinfection, they will not treat the primary lesions.

Oral antifungal agents(choice B) and topical antifungal (choice C) are incorrect because the lesions are not classic for tinea. Tinea generally presents as erythematous bordered plaques with central clearance and superficial scaling. Occasionally, tinea cruris may not have scaling secondary to the moist environment, but the lesions on his thighs and abdomen would have the classic scales.

 

 

An 18-year-old boy with Hodgkin’s lymphoma is admitted to the hospital for his third course of chemotherapy which includes prednisone. On his 6th hospital day, you are called by the nurse to assess a new rash which has slowly progressed to cover the entire anterior and posterior trunk, as well as his arms and part of the face. The nurse is concerned about a possible allergic reaction, secondary to the multiple medications this patient is taking. On examination, you note numerous, small, pinpoint pustules and inflammatory papules of the same stage scattered mostly over his trunk and proximal arms. There are some pustules of the same stage over his forehead as well. The patient denies any significant history of acne prior to the initiation of chemotherapy. At this time the most correct statement about his condition is:

  A. This is acne vulgaris and should be treated with oral minocycline and topical benzoyl peroxide
  B. This is a drug eruption and you should stop all the medications and reinitiate one every week
  C. This is a drug eruption secondary to his chemotherapy and topical corticosteroid will alleviate the problem
  D. This is miliaria secondary to the occlusion effect by his hospital gown
  E. This is steroid acne and topical tretinoin cream may help
Explanation:

The correct answer is E. Acneiform eruptions is correct because it is characterized by papules and pustules resembling acne lesions, not necessarily confined to the usual sites of acne vulgaris. The eruptions are distinguished by their sudden onset. Oral medications such as iodides, bromides, testosterone, cyclosporine, antiepileptic medications, lithium, and systemic corticosteroids are common agents that can lead to acneiform eruption. When medium or high doses of corticosteroids are taken for as short a time as 3-5 days, a distinctive eruption may occur, known as steroid acne. It is a sudden out-cropping of inflamed papules, most numerous on the upper trunk and arms, but also seen on the face. The lesions typically present as papules rather than comedones. Tretinoin cream applied once or twice daily may clear the lesions within 1-3 months, despite the continuation of high doses of corticosteroid.

Acne vulgaris (choice A) is incorrect because of the atypical distribution (proximal arms), and the sudden onset of numerous lesions.

An allergic drug eruption (choice B) is incorrect because typical drug eruptions do not present with pustules. His medication should not be terminated.

Typical drug eruptions do not present with pustules and topical steroids (choice C), especially of the fluorinated types, or when applied under occlusion, may also induce an acneiform eruption and worsen this patient’s skin problem

Miliaria (choice D) is incorrect. This is retention of sweat as a result of the occlusion of eccrine sweat ducts and pores, producing an eruption that is common in patients with prolonged bedrest. These patients typically present with numerous folliculocentric inflammatory papules on the posterior trunk, resulting from extended periods of being on their back. The face, central chest, and arms are typically not occluded in bedrest patients.

 

 

A 28-year-old psychiatric patient jumps out of his bedroom window (2 stories up), landing on his feet. On arrival to the emergency department, he is conscious with no evidence of cranial injury. His blood pressure is 130/80 mm Hg and pulse is 80/min. His chest and abdomen examination are unremarkable, except for tenderness on palpation of bony prominences. Pelvic examination shows no compressive tenderness. Examination of his extremities shows bilateral ankle tenderness. Initial x-rays reveal bilateral calcaneal fractures. This patient is at increased risk for

  A. a femoral shaft fracture
  B. a knee dislocation
  C. lumbar fractures
  D. a pneumothorax
  E. a rib fracture
Explanation:

The correct answer is C. Bones more prone to fracture on landing on feet are the calcaneum and spine. Fractures of the calcaneum and spinal compression fractures need to be excluded in jumpers. Also, posterior hip dislocation and acetabular fractures can result due to forceful movement of the femur in a posterior direction.

Femoral shaft fractures (choice A) happen due to direct injury and can result in massive amounts of bleeding without any external evidence. Patients with femoral shaft fractures are usually hemodynamically unstable due to the massive amount of blood loss.

The knee is a stable joint and dislocation can happen due to direct trauma or due to trauma involving fixation of thigh or leg. A knee dislocation (choice B) is unlikely from landing on feet.

A pneumothorax (choice D) results from direct injury to the chest, however this patient landed on his feet.

Rib fractures (choice E) result from direct injury to the chest. In young adults, ribs are not osteoporotic and hence require significant trauma to fracture.

 

 

You are called by the surgical team to evaluate a new rash on a 55-year-old man who underwent a colectomy for colon carcinoma 10 days prior. He recalls complaining of pressure-induced pain on his buttocks prior to surgery, and the internal medicine nursing staff placed “something” on his back to alleviate the pain. After the surgery, he was transferred to surgical team care. He complains of severe itching and occasional pain on his buttocks. You remove the dressing that is overlying the sacral area and find an erythematous, well-demarcated patch matching the size and shape of the dressing. The surrounding skin is normal and there is no lymphadenopathy. The surgical team is concerned that this is an infectious process. He is afebrile and has been receiving multiple intravenous antibiotics since the surgery. The most appropriate management is to

  A. add an intravenous anti-streptococcal antibiotic to treat erysipelas
  B. perform a punch biopsy to rule out a malignancy
  C. start intravenous immunoglobulin therapy to treat toxic epidermal necrolysis
  D. stop all of his antibiotics and resume one at a time because this is most likely due to a drug eruption
  E. recommend a topical corticosteroid to the area twice a day to treat contact dermatitis
Explanation:

The correct answer is E. Topical steroids are the correct treatment for the irritant contact dermatitis to DuoDerm dressing on this patient. The biggest hint in this vignette is that the rash matches the size and shape of the dressing that was removed from the patient. Often times, irritant dermatitis produces severe pruritus without other symptoms.

Adding IV antibiotics to treat erysipelas (choice A) is incorrect because erysipelas is an acute, inflammatory form of cellulitis that has prominent lymphatic involvement (streaking).

Performing a punch biopsy (choice B) is incorrect, again, because the clinical exam is classic for contact dermatitis. Cutaneous metastasis can present in various shapes and forms, but is much less common.

Starting immunoglobulin therapy (choice C) is incorrect because this patient has no fever or oral or conjunctival mucosal involvement to suggest toxic epidermal necrolysis secondary to medications.

Stopping all of antibiotics (choice D) is incorrect, because the typical antibiotic-induced drug eruption is not limited to one local area with well-demarcated borders.

 

 

A previously healthy 26-year-old office manager comes to the clinic complaining of a widespread rash present for several days. She states she had a large, oval, scaly area on her abdomen arise 2 weeks ago. She treated it with an over-the-counter topical antifungal cream that seemed to be helping, although she says that original rash is still somewhat there. Two days ago a multitude of smaller scaly oval plaques appeared over her “entire body.” She states the eruption is asymptomatic except for the appearance. She lives with her 2 children, ages 3 and 5, and their dog. She was in a monogamous sexual relationship with a man, but that ended 8 months ago and she has not had any sexual contact since that time. She is healthy and does not take any medications. Physical examination shows an oval plaque measuring about 4 cm by 7 cm with a prominent collarette of scale on her left mid abdomen. Many smaller but similar appearing oval plaques measuring about 1-2 cm are present, scattered on most of her torso. On the back these are arranged in a “christmas tree” pattern. She is worried about the rash’s appearance and would like treatment as her family is coming to visit her in 1 week. The next most appropriate course of action is to

  A. obtain a RPR or VDRL and while awaiting the results administer 2.4 million units of penicillin intramuscularly
  B. prescribe ketoconazole lotion, telling the patient to apply it each night to her torso for 1 week and then weekly as prophylaxis
  C. prescribe oral fluconazole and tell her to bathe her dog in antifungal shampoo
  D. prescribe a tapering course of prednisone, but warn her that as her condition is thought to be inherited it may come back at some point in her life.
  E. reassure her about the benign and self-limited nature of her condition and that it will likely resolve in several weeks.
Explanation:

The correct answer is E. This case describes a classic history and examination for pityriasis rosea. This condition usually affects children and young adults, typically in the spring or fall, seems to occur in epidemics, and while the etiological agent is still unknown, is almost assuredly a viral infection. The eruption is characterized by the appearance of a “herald patch,” which is a large and scaly, typically oval plaque on the torso. Occasionally the patients will describe a mild upper respiratory infection or mild flu-like symptoms around the time the herald patch arises. Several weeks later “daughter lesions” appear on the torso which are like smaller versions of the herald patch. On the back these typically follow the lines of skin cleavage and hence resemble a “Christmas tree.” The disease is self-limited in almost all cases and will spontaneously resolve in several weeks after the appearance of the daughter lesions. No treatment is known to hasten recovery.

The eruption of pityriasis rosea and the rash of secondary syphilis can be very similar. Thus checking the patient for syphilis (choice A) is certainly reasonable if the patient has had unprotected sexual contact within the past 6 months. If patient adherence to follow-up is a concern, immediate initiation of treatment is also recommended. In this situation, however, it is unlikely the patient has syphilis. Secondary syphilis does not typically have a “herald patch” and, unless one is immunocompromised, secondary syphilis occurs within 6 months of infection (usually within the first 3 months). The patient in this vignette states her last sexual contact was over 8 months ago.

Ketoconazole lotion (choice B) is effective treatment and prophylaxis for tinea versicolor, also called pityriasis versicolor. It is a superficial yeast infection of the skin, almost always presenting during the summer months. It is characterized by a multitude of small scaly patches on the upper torso and occasionally proximal arms. Confirming the diagnosis can best be done by examining a KOH preparation looking for yeast. Tinea versicolor is not associated with a herald patch. Having the initial lesion on the abdomen and for it to initially present in November would also be extremely usual.

Prescribing a 1-week course of oral fluconazole (choice C) may be a reasonable course of action if the patient had widespread tinea corporis (fungal infection of the skin). Tinea corporis typically presents as a single roughly circular plaque with prominent scale. This scale is usually accentuated at the periphery. Left untreated it usually slowly grows in diameter. Adults typically obtain tinea corporis from children or pets, thus asking that the family dog be washed in antifungal shampoo would be sensible if the pet had any evidence of fungal infection. Fungal infection could be confirmed by examining a KOH preparation looking for hyphea. Widespread fungal infections without any known predisposing factor such as diabetes mellitus can be a clue to immunocompromised states and thus, depending on the situation, testing for HIV may be indicated.

Occasionally pityriasis rosea can be misdiagnosed as psoriasis. Psoriasis is typically comprised of erythematous plaques on the extremities and torso with abundant silvery scale. There is a genetic component to the disease and lifelong recurrences are the norm. In this vignette if you prescribe oral steroids (choice D) you will likely exacerbate the eruption of pityriasis rosea and thus it is clearly contraindicated. In addition, if you suspect the patient has psoriasis, oral steroids may result in dramatic improvement of the rash but almost without exception after the steroids are stopped a “rebound” of the psoriasis occurs and it comes back far more aggressively. It may even trigger pustular psoriasis which can be life threatening.

 

 

A 72-year-old man comes to the office because his wife has been nagging him about “ugly growths” on his hands, forehead, and ears. He says that he began to notice them a few months ago, but lately they have become even more “unsightly”. Now that he is retired, he spends most of his free time on his boat, which he calls his “baby”. He says that he knows that he should use sunscreen, but “who remembers.” He is very suntanned. Physical examination shows multiple scaly, erythematous macules and papules, ranging from 0.1-cm to 0.8-cm. A couple of the lesions on his ears and forehead have a curved horn-like appearance. The most likely diagnosis is multiple

  A. actinic keratoses
  B. basal cell carcinomas
  C. keratoacanthomas
  D. seborrheic keratoses
  E. squamous cell carcinomas
Explanation:

The correct answer is A. This patient most likely has multiple actinic keratoses (AK), which are common lesions seen on sun-exposed skin. AKs are a cutaneous dysplasia of epidermis that may undergo malignant transformation. They typically occur in middle-aged and elderly adults. AKs can be macules or papules, and often have a hyperkeratotic, adherent scale. The “cutaneous horn” is caused by an abnormal growth pattern of the cells within the AK, that resembles an animal horn. The treatment includes cryosurgery, surgical excision, pathologic evaluation, and topical therapy.

Basal cell carcinomas (choice B) are typically characterized by a “pearly papule”, or a papule with central ulceration and a scaly, “pearly”, rolled border.

A keratoacanthoma (choice C) is a rapidly growing nodule that has keratinous debris in the center. It is related to squamous-cell carcinoma.

Seborrheic keratoses (choice D) are benign lesions that can range from tan to dark brown, and usually have a waxy “stuck-on” appearance. They are typically found on the back and face.

Squamous cell carcinomas (choice E) are malignant neoplasms that usually occur on sun-exposed areas of the body, especially on the ears and lower lip. They often appear as a hyperkeratotic, crusted, and ulcerated plaques.

 

 

A 45-year-old woman is in the hospital following a major motor vehicle accident. She sustained fractures of her left femur and left tibia. She feels well after her open reduction and internal fixation of her fractures. On her third hospital day, she mentions that she has pain in her jaw. Since her hospitalization, she has had difficulty opening her mouth completely and when she does, she often hears a “popping” sound. Physical examination reveals moderate pain with palpation of the mastication muscles bilaterally. She is able to slowly open her mouth completely. There are no facial lacerations or bruising. At the time of admission, the patient had a CT of the head that was normal. The most appropriate step at this time is

  A. an arthrography of the temporomandibular joints
  B. a CT of the temporomandibular joints
  C. an MRI of the temporomandibular joints
  D. to recommend a soft diet to limit chewing
  E. to repeat the CT of the head
Explanation:

The correct answer is C. The signs and symptoms of jaw pain, limitation of mouth excursion, and “popping” during mouth opening are most consistent with temporomandibular joint (TMJ) derangement or dislocation. Although most cases are idiopathic, there are predisposing factors such as trauma, dentition problems, joint conditions, and hyperactivity of the muscles of mastication as with bruxism. An MRI is the best diagnostic tool for evaluating all the structures of the TMJ. The disk of the TMJ most commonly dislocates anteriorly.

An arthrography (choice A) is accurate in the diagnosis of internal derangements of the temporomandibular joint (TMJ). It is not frequently performed anymore with the advent of MRI. Arthrography is an invasive procedure associated with complications and patient discomfort that are readily avoided with MRI.

A CT (choice B) is not reliable in the diagnosis of temporomandibular joint (TMJ) dislocation because the disk is inconsistently visualized. It is a good imaging modality to evaluate the bony structures of the jaw, but it is limited in its resolution of the disk, articular cartilage, and tendino-ligamentous insertions.

Recommending a soft diet to limit chewing (choice D) may help the patient avoid pain. It does not, however, address the underlying issue of a probable temporomandibular joint (TMJ) dislocation. An MRI of the TMJ is the most appropriate way to make this diagnosis.

Repeating the CT of the head (choice E) is not indicated in this patient. Her symptoms are limited to the temporomandibular joints (TMJ) and there is no reason to suspect an intracranial process as an etiology. A CT of the head does not include the TMJ in the scan. An MRI of the TMJ is the most appropriate way to make the diagnosis of internal derangements.

 

 

A 12-year-old boy with asthma is brought to the emergency department by his mother because of intermittent right hip pain for the last 2 weeks. The pain is non-radiating and worse with activity. It has now become more constant, worse with weight bearing, and over-the-counter analgesics only give minimal relief. There is no history of night pain and he denies any recent trauma, weight change, or any constitutional symptoms such as fever, chills, or night sweats. He tells you that he went to his pediatrician’s office 10 days ago for the same hip pain and he was told that his physical examination and laboratory studies, including a complete blood count and erythrocyte sedimentation rate, were unremarkable. The pediatrician’s diagnosis was a “pulled muscle or tendon” in the right hip region and he was advised to rest. Now in the hospital, his physical examination shows an obese patient, a limping gait, a leg length discrepancy of 0.5 cm, and intact motor-sensory examination of lower extremities bilateral. The right hip region has intact skin and no focal tenderness to palpation. Passive range of motion of the right hip is decreased on internal rotation. When the hip is flexed, the thigh externally rotates. The most appropriate next step in management is to

  A. admit him to the hospital for intravenous antibiotic therapy, non-weight bearing right lower extremity, and obtain an orthopaedic consultation
  B. recommend immediate crutch walking and non-weight bearing of right lower extremity and obtain pelvic x-rays
  C. repeat complete blood count and erythrocyte sedimentation rate; if the results are normal then send him home with 1 week follow-up in the office
  D. send him home with a prescription for a 0.5 cm shoe insert and physical therapy
  E. send him home with a 10-day course of oral antibiotics with a 2-week follow-up in the office
Explanation:

The correct answer is B. A skeletally immature and obese 12-year-old patient with this history of hip pain is most likely to have a slipped capital femoral epiphysis (SCFE). This growth plate condition does not occur after skeletal maturity, and for males, maturity is on average at 16 years of age. The obligatory external hip rotation with hip flexion is a very common finding in a slipped epiphysis. Immediate non-weight bearing protects the femoral epiphysis from further injury and potential osteonecrosis. Common x-ray findings are consistent with a slippage of the femoral epiphysis. The slippage has been compared to a scoop of ice cream that has partially slipped from the cone. A double density sign and a break in Klein’s line are two x-ray findings. Treatment requires internal fixation of the epiphysis with long screws. The risks of delayed or non-treatment are further slippage and osteonecrosis.

An orthopaedic consult (choice A) will result in discontinuance of the antibiotics and ordering pelvic x-rays. Non-weight bearing is correct and protective, but antibiotics are not indicated because this is not an infectious process.

This patient does not have transient synovitis or any other infectious or inflammatory processes.Therefore, repeating a complete blood count and erythrocyte sedimentation rate, and sending him home if the results are normal with a one week follow up is not necessary(choice C) is incorrect. Again, a delay in diagnosis will result in significant morbidity.

Leg length discrepancy is very common, and less than 2 cm does not typically require treatment. Patients are very often misdiagnosed as having muscle strains or tendon injuries. This delay in diagnosis results in significant morbidity. Sending him home with a prescription for a 0.5 cm shoe insert and physical therapy (choice D) does not address the problem.

This patient does not have an infection. Therefore, antibiotics and discharge to home (choice E) will only delay the correct diagnosis.

 

 

A 34-year-old man comes to the office because of a 5-month history of a left-sided headache and jaw pain. The headache usually feels dull, achy and radiates to his ear. He says that it almost feels as if his “jaw is off-centered”, and it often “clicks” when he eats or chews gums. There is a constant feeling of jaw stiffness, and it feels as if it is “sticking” when he tries to open his mouth. He denies any fever, shortness of breath, changes in weight or vision, or any other arthralgias, and states that he is otherwise in good health. His temperature is 37 C (98.6 F), blood pressure is 115/80 mm Hg, pulse is 65/min, and respirations are 12/min. Laboratory studies show:

The most appropriate next step to establish a diagnosis is to

  A. administer corticosteroids immediately
  B. auscultate the left carotid artery
  C. palpate the area just in front of the left tragus
  D. palpate the left posterior auricular region
  E. palpate the left temporal artery
Explanation:

The correct answer is C. This patient most likely has a temporomandibular joint (TMJ) disorder, which is a very common disorder that can usually be detected by palpating the area just in front of the tragus. Joint clicking may be found when the patient opens and closes his mouth. Symptoms of TMJ include orofacial pain, a noisy joint, and restricted jaw function.

Corticosteroids (choice A) are necessary in cases of temporal arteritis, when the patient complains of visual changes. However, this patient is a young man who does not complain of a headache over the temporal artery or a change in vision. He is afebrile, has a normal hemoglobin, hematocrit, and erythrocyte sedimentation rate, and does not have any systemic symptoms. It is unlikely that he has temporal arteritis.

If a transient ischemic attack is suspected, auscultation of the left carotid artery (choice B), is appropriate to detect the presence of a carotid bruit. However, this young patient is describing the 3 main symptoms of TMJ which include orofacial pain, a noisy joint, and restricted jaw function. It is unlikely that he has carotid artery disease.

This patient most likely has a temporomandibular joint (TMJ) disorder, which is a very common disorder that can usually be detected by palpating the area just in front of the tragus, not the left posterior auricular region (choice D). Here is a practical explanation: put your finger in front of your ear, and open and close your mouth. You can feel the movement of your jaw. Now, put your finger behind your ear, in the posterior auricular region, and open and close your mouth. You cannot really feel your jaw move.

If temporal arteritis was suspected, particular attention should be given to the left temporal artery (choice E). However, this patient is a young man who does not complain of a headache over the temporal artery or a change in vision, is afebrile, has a normal hemoglobin, hematocrit, erythrocyte sedimentation rate, and does not have any systemic symptoms. Palpation of the temporal artery is appropriate during the examination to rule out temporal arteritis, but particular attention should be given to the area just in front of the left tragus, to detect TMJ.

 

 

A 21-year-old college football player comes to the university student health center clinic because of pain and swelling of his right knee for the past 2 weeks. He says the pain started after he was tackled during football practice 2 weeks ago. Initially, the pain was severe and he thought he heard a “pop” right after he was tackled. He was not able to walk immediately after the injury, but he has been walking normally for the past week. An x-ray of the right knee on the day of the injury was normal. On physical examination, there is a small effusion in the right knee. There is no erythema or focal areas of tenderness. He has full range of motion in the right knee and has a positive anterior drawers test. The most appropriate next step is to

  A. order a CT of the knee
  B. order an MRI of the knee
  C. perform arthrocentesis
  D. prepare him for arthroscopy
  E. repeat x-ray of the knee
Explanation:

The correct answer is B. The presentation of knee pain following trauma with an associated “pop” and the immediate inability to ambulate is highly suspicious for a rupture of the anterior cruciate ligament (ACL). The ACL is frequently injured in contact sports such as football. Patients will also often complain of episodes of instability or giving-way after the initial symptoms of pain and swelling resolve. The anterior drawer test involves firmly pulling the lower leg above the calf forward to test for laxity. If the ACL is ruptured, the lower leg is able to move anterior to the femur. The most appropriate study to diagnose ACL injury is an MRI. An MRI is the best imaging modality to study the soft tissues, ligaments, and menisci of the knee. When torn, the ACL is most often simply not visualized. Sometimes, the actual disruption is seen. A partial tear of the ACL will demonstrate an abnormally high signal in the ligament. An MRI can also detect associated injuries such as a medial meniscus tear.

A CT of the knee (choice A) is not a useful study for this patient. A CT is very good in the evaluation of the bones of the knee and may demonstrate fractures not detected on the x-ray. It is not a good diagnostic tool in the evaluation of the ligaments, tendons, and menisci of the knee or other joints.

Arthrocentesis (choice C) is a procedure used to obtain synovial fluid for further analysis. This procedure is used to diagnose infections and crystalline disease of the joints. Although the patient has a knee effusion, it is not necessary to analyze the synovial fluid in his particular presentation. The most likely etiology of the effusion is an inflammatory reaction related to his primary diagnosis of an anterior cruciate ligament injury.

Arthroscopy (choice D) is necessary to repair the ruptured anterior cruciate ligament (ACL). However, it is not the most appropriate procedure to order at this time. It is critical to obtain an MRI of the right knee, to not only confirm a suspected ACL tear, but to also evaluate for other concomitant injuries to the other ligaments and menisci of the knee. Traumatic injury to the ACL is often associated with tears in the medial meniscus and the medial collateral ligament. An MRI of the knee will provide a road map for orthopedic surgeons during the therapeutic arthroscopic procedure.

A repeat x-ray of the knee (choice E) is not a useful study for this patient. The initial x-ray was normal, and the patient’s symptoms have not changed since this study. A plain x-ray of the knee is good for evaluating the bones and for an effusion. It is useful initially for demonstrating fractures that are highly suggestive of internal derangements in the knee. A Segond fracture is a tiny avulsion off the posterior lateral tibial joint line and is almost always associated with a tear of the anterior cruciate ligament. An x-ray is not a good diagnostic tool in the evaluation of the ligaments, tendons, and menisci of the knee or other joints.

 

 

A 51-year-old woman comes to the office because of a “lump” in her left axilla. She says that she noticed it 3 months ago in the shower and it has been slowly increasing in size. She has not seen a physician in ages because her mother died from ovarian cancer at age 42 and she is angry at “all physicians” because they did not find it before it was too late. She is generally very healthy, exercises regularly, eats a low-fat diet, does not smoke cigarettes, lives alone with her dog, and has not traveled recently. She is allergic to flowers and cats. She denies fever, night sweats, or fatigue. Physical examination shows a 2.0 cm fixed round lesion in her axilla. The remainder of the examination, including a clinical breast examination and pelvic examination, are normal. A complete blood count is unremarkable. The most appropriate next step is to

  A. obtain cultures for Epstein-Barr virus and Cytomegalovirus
  B. order a complete biochemical profile and chest x-ray
  C. reassure her and have her return in 2 weeks for reexamination
  D. schedule a mammography
  E. send her to a surgeon for a lymph node biopsy
Explanation:

The correct answer is D. In general, a patient with axillary adenopathy, without fever, weight loss, fatigue, and night sweats should be evaluated for cellulitis, cat-scratch disease, sporotrichosis, and breast cancer. Since the first 3 disorders can basically be ruled out with a complete history and physical examination, a mammography is needed to evaluate for breast cancer.

Epstein-Barr virus and Cytomegalovirus (choice A) are typically associated with cervical adenopathy, not a single, fixed axillary lymph node.

A biochemical profile and chest x-ray (choice B) are not indicated at this time in this patient with a single fixed axillary lymph node, since she has a family history of cancer and has not been to a doctor in “ages” and is probably due for a mammogram.

Since this lymph node has been present for months, it is inappropriate to reassure her and have her return in 2 weeks for reexamination (choice C) because it is unlikely to decrease in size spontaneously by then. She requires evaluation at this time.

A lymph node biopsy (choice E) would be the appropriate step if this was a supraclavicular lymph node. However, the evaluation of an axillary lymph node in a woman typically includes a mammogram with a biopsy if a lesion is found. A lymph node biopsy should be performed after a breast lesion is ruled out.

 

 

A 61-year-old woman is admitted to the hospital for fatigue and pain in her hands and arms. She has a long history of rheumatoid arthritis and has had multiple surgical procedures to correct her upper extremity deformities. Over the past few days, she has had an exacerbation of her disease such that she has been unable to perform her activities of daily living. She complains of profound fatigue, fever, and a recent 10 pound weight loss in one month. She has no tobacco or ethanol history. Her medications currently include gold, methotrexate, and thiazide daily. Her temperature is 38.0 C (99.4F), blood pressure is 130/55 mm Hg, pulse is 92/min, and respirations are 14/min. She has bilateral ulnar deviation and a number of 2-3 cm subcutaneous nodules across her fingers and elbows. Her lungs are clear, but her spleen is palpable in the left upper quadrant. Chest radiograph reveals multiple 1 cm pulmonary nodules in both lung fields. The most appropriate next step in management is to

  A. determine rheumatoid factor level
  B. increase the dose of methotrexate
  C. initiate cyclophosphamide therapy
  D. order radiograph of the hands
  E. schedule an open lung biopsy
Explanation:

The correct answer is C. The patient has severe rheumatoid arthritis. She has all of the classical findings, including morning stiffness, polyarticular involvement, and rheumatoid nodules. The etiology of RA is unknown but it is a progressive, destructive disease. The disease is seen in women at least twice as often as in men. It has a prevalence of nearly 10% worldwide in persons over the age of 65. Her radiographic findings indicate rheumatoid nodules (Caplan syndrome) and her splenomegaly, coupled with likely neutropenia, is Felty syndrome seen with severe RA. For these patients, once they have failed traditional management strategies, immunosuppressive agents, such as cyclophosphamide or azathioprine, are indicated.

A rheumatoid factor level (choice A) will not be of any value in managing this patient. Although RF levels often correlate with disease severity, they have no prognostic significance nor do they reflect efficacy of therapy.

Increasing the dose of methotrexate (choice B) will likely have minimal effect on the course of this patient’s disease. Given her recent exacerbation and evidence on physical examination of severe disease, a more aggressive approach to treatment is warranted for this woman.

A radiograph of the hands (choice D) is not needed to evaluate this patient’s findings. She has classic ulnar deviation and rheumatoid nodules. A radiograph will only disclose additional likely bony deficits secondary to the destruction of bone and cartilage from her RA process.

Although the nodules, coupled with the fatigue and weight loss could suggest cancer, an open lung biopsy (choice E) is not the appropriate diagnostic step to take for evaluation of such symptoms. A CT scan of the thorax would precede such an invasive option. However, this patient has no risk factors for lung cancer and has nodules in association with severe RA, making Caplan syndrome the most likely diagnosis.

 

 

A 10-year-old African American boy is brought into the office by his mother because of concerns over “white spots” on his skin that are increasing in size and number. His mother reports the first spot began 2-3 years ago on his right index finger, which was stable until recently when it increased in size. Of more concern to the mother was the increase in number of similar lesions of various size around his mouth, anus, and penile tip. Further questioning reveals a family history of diabetes and thyroid disease. Full skin examination shows depigmented, flat patches that are well demarcated with no scales or surrounding erythema located at the perioral, perianal, tip of penis, and bilateral knees as well as right index and middle fingers. Management of this patient should include

  A. complete blood count and chemotherapy
  B. ketoconazole shampoo 3 times per week and oral ketoconazole for 2 weeks
  C. sunscreen and sun avoidance
  D. topical antifungal cream
  E. topical corticosteroids and phototherapy
Explanation:

The correct answer is E. Topical steroid and phototherapy are the two main therapeutic modalities in treating vitiligo. Vitiligo is an acquired disorder of depigmentation characterized by loss of melanocytes from the epidermis, the mucous membranes, and other tissues. A vitiliginous lesion usually presents as either a hypopigmented or snow-white, well circumscribed macule or patch. In most cases, a single lesion is initially noticed and new lesions can develop either immediately or over an extended period of time. Lesions progress in size and number. The most common sites are the face, hands, legs, glans of the penis, and the perianal region. Thyroid diseases (both hypo- and hyperthyroidism as well as Graves disease), diabetes mellitus, pernicious anemia, Addison disease, multiglandular insufficiency syndrome, and alopecia areata have been associated with vitiligo.

Complete blood count and chemotherapy (choice A) is appropriate if you suspect cutaneous T cell lymphoma. It is a chronic, frequently fatal disease of helper T cells commonly seen in patients in the fifth to seventh decades of life. Clinically, the lesions can present as erythematous, pruritic, brown-to-purple indurated plaques with central clearing or multiple, round, dome-shaped tumors. Pruritus is usually severe and excoriations are prominent.

Topical and oral ketoconazole (choice B) is incorrect because these are the mainstay of therapy in tinea versicolor. It is an extremely common superficial fungal disorder characterized by multiple scaling, oval, macular, and patchy lesions usually distributed over the upper portions of the trunk, proximal arms, and occasionally the face. The lesions are hypopigmented or hyperpigmented, caused by Pityrosporum orbiculare. Generally asymptomatic, the lesions become lighter than the surrounding skin in the summer and relatively darker during winter, hence the name tinea versicolor.

Sunscreen and sun avoidance (choice C) is incorrect because these are treatments for pityriasis alba. This is a common disorder in children, characterized by discrete asymptomatic hypopigmented patches on the face, neck, upper trunk, and proximal extremities. Most cases appear following sun exposure and result from a disturbance in pigmentation of the affected areas.

Topical antifungal cream (choice D) is the treatment for tinea corporis (ringworm), which is characterized by one or more annular, sharply circumscribed scaly patches with a clear center and scaly vesicular, papular, or pustular border. The disorder is most commonly seen in children and in individuals in warm, humid climates.

 

 

A worried mother brings in her 7-year-old son who recently returned from summer camp with a new “itchy” rash on and around his umbilicus. There is a similar rash on his left wrist, under his brand new metal watch that he received for his seventh birthday before he left for the camp. His temperature is 37.0 C (98.6 F). He opens his jeans to show you a well-demarcated, erythematous, circular plaque with numerous small vesicles at the periumbilical area. The surrounding skin is normal without xerosis. There is a circular, well demarcated erythematous plaque with similar vesicles on his left wrist. Oral and conjunctival mucosa, as well as the remainder of the cutaneous examination is unremarkable. The boy appears happy but the mother is very concerned. At this time the most appropriate next step is to

  A. explain that this is a drug allergy and question her further about new medications
  B. explain that this is a nickel allergy and that he should avoid metals in clothing and jewelry that contain mixtures of nickel
  C. reassure her and tell her Molluscum contagiosum often resolves without treatment in children
  D. tell her that this is childhood eczema and that he will most likely require life-long treatment
  E. tell her that this is a viral exanthem which will resolve spontaneously
Explanation:

The correct answer is B. Nickel is the most common cause of allergic contact dermatitis. The critical factor is the amount of nickel released from metals in intimate contact with the skin. Sweat, containing sodium chloride, may exacerbate the dermatitis in persons who sweat profusely. The factors in this setting, which include the jeans (metal buttons), his watch with a metal base, and the hot summer season, are important clues to nickel contact dermatitis. Therapy includes avoidance of metal to skin (wear pants with elastic bands) and the application of topical corticosteroids to affected areas.

Drug allergy (choice A) is incorrect because usually drug reactions result in generalized morbilliform, erythematous papules that begin on trunk and extend peripherally. No culprit medication was given in the history and this patient has localized, well-demarcated lesions that are inconsistent with drug eruption.

Molluscum contagiosum (choice C) is incorrect because poxvirus infections tend to be grouped, flesh-colored, or pearly papules a few millimeters in diameter. As they progress, they develop a distinct central depression (umbilication) and a white, curd-like core may easily be expressed. The lesions are generally asymptomatic.

Eczema or atopic dermatitis (choice D) is incorrect because atopic dermatitis patients have generalized xerosis and tend to worsen in the winter months. Sites of predilection in children are antecubital and popliteal fossa, face, and neck. Other findings in these patients are keratosis pilaris, accentuated palmar creases, lichenification, cataracts, and “allergic shiners” (infraorbital discoloration).

A viral exanthem (choice E) is incorrect because viral exanthems tend to be generalized and, depending on the virus, can have associated systemic findings such as upper respiratory symptoms, fever, cough, and oral mucosa involvement.

 

 

A 43-year-old female former nurse comes to the clinic complaining of a rash around a healing laceration on her left dorsal forearm. She reports that she cut her forearm with a knife while removing it from her dishwasher 5 days ago. The cut was not deep and she did not seek medical attention. She has been cleaning it with hydrogen peroxide and applying neomycin ointment, followed by wrapping it in a bandage twice a day. Yesterday evening during the dressing change she noted some pruritus and erythema in the area of the wound; this morning she was alarmed to find the area extremely itchy and with a “horrible rash.” She denies fevers or chills and states that besides the situation with her wrist, she feels well. Looking at her chart and speaking with her you find her only medication is lisinopril for hypertension. She has no other medical problems of which she is aware. Physical examination of the left forearm reveals a shallow, healing, 2.5 cm long laceration. Extending approximately 2 cm from the wound in each direction is erythema and minute vesicles filled with clear fluid. There is no lymphadenopathy and her temperature is normal. The most appropriate management is to

  A. prescribe acyclovir ointment
  B. prescribe acyclovir tablets
  C. prescribe cephalexin tablets
  D. prescribe hydrocortisone ointment
  E. recommend bacitracin ointment
Explanation:

The correct answer is D. The appearance of the skin, the history of applying neomycin ointment, and the time course all point to an allergic contact dermatitis. Neomycin is a very common sensitizer, with as many as 5% of the population being allergic to the compound in some studies. Treatment for mild cases of allergic contact dermatitis is stopping the offending agent and applying topical steroids. This case also highlights the fact that many people wrongly assume that topical products, especially over-the-counter products, are not medicines.

Acyclovir ointment (choice A) has been shown to be marginally effective for speeding healing of herpes labialis, but has not been studied for extralabial sites. For a large area of herpes vesicles, oral acyclovir would be the preferred treatment.

Acyclovir tablets (choice B) would be reasonable if this was a herpetic outbreak. The classic appearance of herpes is grouped vesicles on an erythematous patch or edematous plaque. The localization of herpetic lesions to the forearm would be unusual, as would the subjective symptom of pruritus since herpetic lesions are usually painful. Lack of lymphadenopathy would be another factor that would militate against a diagnosis of herpes. Definitive diagnosis of herpes would be either positive culture or identifying multinucleated cells on a Tzanck prep.

Cephalexin (choice C) is a treatment for impetigo, particularly if it is widespread. Also, if you suspected the patient had cellulitis, systemic antibiotics would be indicated. Clinically cellulitis would have erythema around the wound, but fever and/or lymphadenopathy would be expected. In addition, cellulitis does not present with vesicles nor with pruritus.

Bacitracin ointment (choice E) may be a reasonable choice if you suspected impetigo, but that entity consists of erythematous plaques with a prominent scale-crust. This crust is classically described as honey-colored. Impetigo generally does not exhibit vesicles. It would also be uncommon to develop in an area being cleaned and having antibacterial ointment applied twice a day. Note, the lack of fever and the lack of lymphadenopathy are entirely consistent with impetigo. Another reason bacitracin would be a poor choice in this situation is a high percentage of patients with contact sensitivity to neomycin will cross-react to bacitracin, thereby aggravating the contact dermatitis.

 

 

A 32-year-old woman comes to the office because of “hair loss.” Even though she has not noticed much of a difference, her friends have been telling her that her “gorgeous thick hair seems to be getting a bit sparse” lately. She states that she only notices the difference when she looks at pictures of herself that were taken 1 years ago. She is generally healthy and has no other complaints. She exercises 5 to 6 days a week, eats a “low fat, low protein” diet, does not smoke cigarettes, and drinks a glass of red wine with dinner. Her temperature is 37.0 C (98.6 F), blood pressure is 110/70 mm Hg, and pulse is 55/min. Physical examination shows diffuse thinning of the hair and a visible scalp. At this time, the most correct statement about her condition is:

  A. The diffuse hair is unrelated to trichotillomania
  B. The diffuse hair loss is unrelated to her “low fat, low protein” diet
  C. Thyroid dysfunction is not the cause of her diffuse hair loss
  D. She has an autoimmune condition called alopecia areata, in which lymphocytes attack the hair follicle
  E. She should discontinue her vigorous exercise program and the hair will regrow
Explanation:

The correct answer is A. Trichotillomania is a form of obsessive compulsive disorder where individuals pull out large amounts of their own hair. It is usually concentrated in certain areas, particularly above the ear (because this may be the most convenient area to pull), and leads to patchy hair loss. Diffuse thinning of the hair is not the typical pattern.

It is incorrect at this time to say that her diffuse hair loss is unrelated to her “low fat, low protein” diet (choice B) because diffuse thinning of the hair is associated with many vitamin and mineral deficiencies such as zinc, iron, and biotin deficiency. It is also seen in some individuals with protein deficiency.

Hyperthyroidism and hypothyroidism (choice C) are associated with diffuse thinning of the hair. Many individuals with thyroid dysfunction are asymptomatic, and therefore, you cannot conclude that she does not have thyroid dysfunction until you do the appropriate thyroid tests (thyroid stimulating hormone levels).

Alopecia areata is an autoimmune condition where lymphocytes attack the hair follicle (choice D) and is typically associated with well-circumscribed areas of hair loss, not diffuse thinning.

At this time it is not appropriate to tell her that her hair will grow back if she discontinues her vigorous exercise program (choice E). Her diffuse hair loss can be due to many different things, such as vitamin and mineral deficiencies, protein deficiency, thyroid dysfunction, and HIV infection. It is possible that the exercise is associated with her problem, but at this time, before you evaluate the other conditions, it is not appropriate to draw this conclusion.

 

 

A 33-year-old man comes to the urgent care clinic with worsening pain of the right knee for 2 weeks. He is an active tennis player, but cannot recall a history of trauma. He is normally very healthy. His temperature is 37.0 C (98.6 F). A focused physical examination confined to the musculoskeletal system is performed. The right knee is erythematous and tender to palpation and an effusion is detected. There is normal motion of the knee and drawer signs are negative. The remainder of the joints are normal. The next step in management of this patient is

  A. aspiration of the knee joint
  B. CBC and blood culture
  C. lyme titers
  D. MRI of the knee
  E. a serological test for rheumatoid factor
Explanation:

The correct answer is A. Monoarticular joint pain in a young person is infection until proven otherwise. A complete sexual history and GU exam are essential to evaluate for gonococcal arthritis. Given the high morbidity of an infected joint, aspiration should be performed urgently. This result will guide antibiotic therapy. Less likely entities include internal derangement, such as a meniscus tear or the initial presentation of an arthritis.

Evaluation with systemic labs such as a CBC and blood culture (choice B) is likely to be negative given lack of pyrexia.

Lyme disease (choice C) may present with a migratory polyarthritis in patients from the New England region and other infested areas. There is usually a history of outdoor activity allowing the patient to become bitten by the Ixodes tick which transfers the spirochete Borrelia burgdorferi to the patient. In cases of Lyme disease, the rash of erythema migrans is often noticed several weeks or even months before arthritic symptoms, typically, symptoms of the knee are noted. In this case, a rapidly destructive bacterial infection of the joint must be excluded long before Lyme titer results return from the laboratory.

An MRI of the knee (choice D) is useful to exclude internal derangement such as a ligamentous sprain or tear, meniscal injury, fracture, or contusion. Given the erythema of the knee, but a lack of motion pain or limitation, infection is much more likely. A knee effusion or bone edema on MRI would suggest infection, but arthrocentesis is necessary immediately.

Serological evaluation for arthritis (choice E) such as rheumatoid factor or HLA-B27 positivity should be performed once infection is excluded with an arthrocentesis and internal derangement is excluded with an MRI.

 

 

A 72-year-old man comes to the office because of “rectal pain and bleeding” for the past few weeks. The pain is relatively constant and is not associated with defecation. The bleeding is intermittent and he thinks that it is present most of the time because he finds bright red blood on his undergarments. He has also been feeling a “bit weak” lately and has lost 10 pounds, but he attributes this to “getting old.” He denies any change in bowel habits. He recently moved to your town to live with his daughter and he tells you that he has a history of hemorrhoids and that he has been eating a high fiber diet and taking stool softeners, just as the previous physician instructed him to. Physical examination shows a 1.3 cm blue-black partially raised, ulcerated lesion, just above the anal verge. Rectal examination reveals guaiac-negative hard, brown stool. Anoscopy is unremarkable. The most appropriate next step is to

  A. admit him to the hospital for the evaluation for rectal carcinoma
  B. advise him to take sitz baths, apply a topical steroid cream, and increase his fiber intake
  C. incise the lesion, evacuate the thrombus, and apply compression to the incised area
  D. perform an immediate rubber band ligation and tell him that it will undergo degeneration
  E. perform an excisional biopsy of the lesion
  F. prescribe a bowel preparation and schedule a colonoscopy for next week
Explanation:

The correct answer is E. This patient most likely has malignant melanoma. The features that make this lesion suspicious are its blue-black color, partially raised, asymmetrical pattern, and the size (>6 mm). The first part of the diagnosis of any melanoma is by visual inspection. Any lesion suspicious for a melanoma should be biopsied. The preferred method is by excisional biopsy, which is full thickness. Avoidance of shave biopsy, curettage or incisional biopsy is recommended for any lesion thought to be melanoma because it will alter the final histological evaluation of the tumor. While the anus is not one of the most common sites for melanoma, it can occur anywhere and should always be part of your differential diagnosis of a suspicious lesion.

It seems that this patient may have malignant melanoma that should be evaluated with a biopsy and it is therefore unnecessary to admit him to the hospital for the evaluation of a rectal carcinoma (choice A) at this time. He does complain of rectal bleeding, however he has not had a change in bowel habits and his rectal examination and anoscopy were unremarkable. At this time, a biopsy is the best next step.

Sitz baths, application of a topical steroid cream, and increasing his fiber intake (choice B) are the usual treatment for symptomatic external hemorrhoids which often present with rectal pain and bleeding. Physical examination and anoscopy will often show the hemorrhoids. Increased fiber intake and possibly stool softeners are part of the long-term management of hemorrhoids. This patient may have a malignant melanoma and therefore requires evaluation with a biopsy.

Incise the lesion, evacuate the thrombus, and apply compression to the incised area (choice C) is the treatment for a thrombosed external hemorrhoid. Pressure by compression is usually all that is needed to control the bleeding. The typical presentation of a thrombosed external hemorrhoid is an acute onset of very severe perianal pain, particularly when walking and sitting. An external hemorrhoid arises below the dentate line and appears as a tender blue swelling at the anal verge.

Rubber band ligation (choice D) is the treatment for refractory, symptomatic internal hemorrhoids. Local anesthesia is given, an anoscope is inserted, and a rubber band is placed around the hemorrhoid. The hemorrhoid will eventually become necrotic and slough over time. Internal hemorrhoids arise above the dentate line. This patient has a suspicious lesion near the anal verge that requires biopsy evaluation.

Bowel preparation and colonoscopy (choice F) are not the next step in management of a lesion that is suspicious for a malignant melanoma. A biopsy needs to be performed to establish a diagnosis.

 

 

A 12-year-old boy is brought to the office because of a 2-week history of pain and swelling of his right leg. The pain is worse on the shin, just below the knee, and is exacerbated by running, jumping, and going up and down stairs. His mother states that he is a very active child; “he is always on the go.” He plays basketball with friends on the court in their backyard, and baseball on a neighborhood little league team. He recently started playing volleyball in gym class. Physical examination reveals a pubescent boy with point tenderness and swelling over the right tibial tubercle. The left leg is unremarkable. The most likely diagnosis is

  A. “growing pains”
  B. Legg-Calve-Perthes disease
  C. Osgood-Schlatter disease
  D. osteosarcoma
  E. slipped capital femoral epiphysis
Explanation:

The correct answer is C. This patient most likely has Osgood-Schlatter disease. The typical patient is a physically active pubescent boy who has pain and swelling over the tibial tuberosity. The pain is exacerbated by physical activity. It is caused by apophysitis, (inflammation of the tibial tuberosity), and cartilage detachment. Diagnosis is made by history and tibial tuberosity tenderness. Treatment is the reduction of physical activity.

“Growing pains”, (choice A) are usually characterized by deep, severe, bilateral, diffuse pains that are worse at night. There is no associated limp.

Legg-Calve-Perthes disease (choice B), avascular necrosis of the femoral head, is characterized by hip and knee pain, a limp, and decreased range of motion. It usually affects boys between 4 to 8 years of age. Casting and surgery are treatment options.

Osteosarcoma (choice D), is the most common malignant bone tumor in kids. It occurs in the metaphyses of long bones, and presents with pain, swelling, and a palpable mass. X-rays show a lytic lesion with a “sunburst” pattern. Treatment is surgery.

Slipped Capital Femoral Epiphysis (choice E), is a disorder of overweight boys that is caused by a displacement of the femoral head from the femoral neck. There is knee or thigh pain, and a limp. Treatment is immediate surgical fixation.

 

 

A 12-year-old boy is brought to the office because of a 2-week history of pain and swelling of his right leg. The pain is worse on the shin, just below the knee, and is exacerbated by running, jumping, and going up and down stairs. His mother states that he is a very active child; “he is always on the go.” He plays basketball with friends on the court in their backyard, and baseball on a neighborhood little league team. He recently started playing volleyball in gym class. Physical examination reveals a pubescent boy with point tenderness and swelling over the right tibial tubercle. The left leg is unremarkable. The most likely diagnosis is

  A. “growing pains”
  B. Legg-Calve-Perthes disease
  C. Osgood-Schlatter disease
  D. osteosarcoma
  E. slipped capital femoral epiphysis
Explanation:

The correct answer is C. This patient most likely has Osgood-Schlatter disease. The typical patient is a physically active pubescent boy who has pain and swelling over the tibial tuberosity. The pain is exacerbated by physical activity. It is caused by apophysitis, (inflammation of the tibial tuberosity), and cartilage detachment. Diagnosis is made by history and tibial tuberosity tenderness. Treatment is the reduction of physical activity.

“Growing pains”, (choice A) are usually characterized by deep, severe, bilateral, diffuse pains that are worse at night. There is no associated limp.

Legg-Calve-Perthes disease (choice B), avascular necrosis of the femoral head, is characterized by hip and knee pain, a limp, and decreased range of motion. It usually affects boys between 4 to 8 years of age. Casting and surgery are treatment options.

Osteosarcoma (choice D), is the most common malignant bone tumor in kids. It occurs in the metaphyses of long bones, and presents with pain, swelling, and a palpable mass. X-rays show a lytic lesion with a “sunburst” pattern. Treatment is surgery.

Slipped Capital Femoral Epiphysis (choice E), is a disorder of overweight boys that is caused by a displacement of the femoral head from the femoral neck. There is knee or thigh pain, and a limp. Treatment is immediate surgical fixation.

 

 

An 18-year-old woman comes to the clinic because of a 4-month history of a “red rash” on her elbows, knees, and around her “belly button.” She noticed the lesions during the winter, but was not particularly concerned because they were covered up by pants and long sleeves. Now it is summer and she is too embarrassed to wear shorts or a bathing suit. She has no significant past medical history, is up-to-date on her immunizations, and has not traveled recently. She takes no medications and has no known allergies. She tries to avoid all sun exposure because she tends to “burn, not tan.” Physical examination shows erythematous plaques on her elbows, knees, and umbilicus. There is a silvery scale covering the majority of each lesion that bleed when you scrape it. The remainder of the examination is unremarkable. The most appropriate next step is to

  A. advise her to avoid sun exposure, especially direct sunlight on the lesions
  B. biopsy each lesion and send for histologic evaluation
  C. inject each lesion with a medium-potency corticosteroid
  D. prescribe a medium-potency corticosteroid ointment and topical calcipotriene
  E. refer her to a dermatologist
Explanation:

The correct answer is D. This patient most likely has psoriasis, which is a relatively common skin condition that affects approximately 2% of the population. Psoriasis is characterized by erythematous, thickened plaques with a silvery scale. Scraping of the scale may lead to pinpoint bleeding, often called the Auspitz sign. The diagnosis is usually made by physical examination. The treatment for localized lesions typically begins with a topical corticosteroid and topical calcipotriene (a vitamin D analog that enhances normal keratinization and inhibits epidermal cell proliferation). A topical corticosteroid can also be used with a coal tar product, which possibly suppresses epidermal DNA synthesis, as a first-line therapy. These can be tapered if the lesions subside. If this in ineffective, corticosteroid and anthralin or tazarotene therapy combined with ultraviolet phototherapy should be considered. If this is still ineffective, the patient should be sent to a dermatologist.

It is inappropriate to advise her to avoid sun exposure, especially direct sunlight on the lesions (choice A) because sunlight is thought to be helpful in treating psoriasis. She should be encouraged to obtain natural sunlight exposure for a few minutes a day. But she needs to be advised that unaffected areas should be covered with a sunscreen.

While psoriasis can be diagnosed by a biopsy, it is inappropriate to biopsy each lesion and send for histologic evaluation (choice B). The diagnosis can usually be made based on clinical presentation, especially when the lesions are classic, as they are in this case.

It is inappropriate to inject each lesion with a medium-potency corticosteroid (choice C) at this time. This is usually reserved for psoriasis that is not controlled by topical therapy.

It is not necessary to refer her to a dermatologist (choice E) at this time. Any physician should be able to recognize and treat localized psoriasis. If the treatments are not effective in controlling her disease, you should refer her to a dermatologist.

 

 

A 6-year-old boy is brought to the office by his parents who are concerned because he has been refusing to use his left arm for 1 day. The parents report that he has been in good health and has not suffered any recent falls or injuries to the arm that they are aware of. The father does recall one incident 2 days ago when he pulled upward on the boy’s right arm to prevent him from tripping as they descended a flight of stairs. The boy is holding his right arm with the elbow flexed and the forearm pronated. He begins to cry when you attempt to examine the arm. The most appropriate next step is

  A. closed reduction and cast immobilization
  B. a CT of the right arm
  C. measurement of compartment pressure of the forearm
  D. supination of the forearm with the elbow flexed
  E. a trial of compressive bandage on the right arm
Explanation:

The correct answer is D. This boy has the signs and symptoms of a very common injury in little children. “Nursemaid’s elbow” or subluxation of the radial head, which occurs when there is longitudinal traction on the pronated extended elbow. This traction can cause the radial head to slip from beneath the annular ligament. Children will often refuse to move the arm following this injury. X-rays of the forearm and elbow are usually normal and treatment involves supination of the forearm with the elbow flexed. Recurrence is uncommon and no immobilization of the affected arm is necessary.

Closed reduction and cast immobilization (choice A) is necessary for fractures of the forearm, not for subluxation of the radial head. Fractures of the forearm are also very common in children, but usually have an antecedent history of fall or injury.

Obtaining a CT of the right arm (choice B) is not necessary for this patient. X-rays of the right arm can be done to rule out a fracture or hemarthrosis. A CT of the extremity is sometimes performed if the x-ray demonstrates a fracture. The history given by the boy’s parents and the boy’s presenting symptoms strongly suggest subluxation of the radial head. X-rays of the forearm and elbow are usually normal in this setting and are not necessary to make the diagnosis.

There is no reason to suspect compartment syndrome (choice C) in this child. Signs of compartment syndrome include severe pain in the limb especially with active or passive stretching of the ischemic muscle, cyanosis, and numbness. This syndrome is an uncommon complication of any traumatic injury to the extremities. Subluxation of the radial head is not associated with compartment syndrome.

A trial of a compressive bandage (choice E) is not necessary in this patient. Supination of the forearm should reduce the subluxation of the radial head. Compressive bandages or immobilization are not necessary after reduction of the subluxation.

 

 

You are seeing a 23-year-old woman in your office for a follow up visit. She had presented for an initial visit a month ago complaining of swelling in her fingers. Today, she continues to describe edema and erythema of her metacarpophalangeal joints. She has also had some edema of her left elbow. She explains that her morning stiffness is lasting more than 1 hour. She is currently on no medications and has no allergies to medications. Her mother and maternal aunt have a history of severe rheumatoid arthritis. Her temperature is 37.2 C (99.0 F), blood pressure is 123/65 mm Hg, pulse is 76/min, and respirations are 18/min. She has edema and erythema of the metacarpophalangeal joints of both hands. The remainder of her joint examination is unremarkable. Her breath sounds are clear, and her cardiac rhythm is regular. The results of routine laboratory studies sent during her previous visit are consistent with a diagnosis of rheumatoid arthritis. The most appropriate pharmacologic intervention at this time to alter the course of her disease is

  A. aspirin
  B. 5-fluorouracil
  C. methotrexate
  D. penicillamine
  E. sertraline
Explanation:

The correct answer is C. It is reasonable to start the patient on a trial regimen of methotrexate since studies have shown that disease modifying antirheumatic drugs (DMARD) such as methotrexate have improved outcomes with rheumatoid arthritis, and may alter the course of her disease. An NSAID or COX-2 specific inhibitor should also be used.

Aspirin (choice A) is a first-time therapy that can help with the signs and symptoms of her disease but will have minimal effect on the progression of disease.

5-fluorouracil (choice B), an antineoplastic agent, has no proven benefit with rheumatoid arthritis.

Penicillamine (choice D), while being a disease modifying antirheumatic drug (DMARD), is associated with too many toxic side effects to be chosen as a first-line DMARD.

Sertraline (choice E), a selective seratonin reuptake inhibitor (SSRI) used for the treatment of depression, has no role in the management of rheumatoid arthritis.

 

 

A 68-year-old retired firefighter comes to the clinic because of pain in his left knee. Despite being retired, he has remained very active playing tennis and golf 4 times a week. He has been unable to play tennis for the last month because of his knee pain. In fact, he says “sometimes when I’m walking, my knee just locks up and I can’t move it at all!” His past medical history includes hypertension, gout, and a cholecystectomy 10 years ago. His medications include an antihypertensive and an occasional nonsteroidal antiinflammatory drug for gout flares in his ankle. Physical examination reveals full range of motion in the left knee and moderate pain along the medial aspect of the knee. There is a small effusion, but no erythema. Anterior and posterior drawer tests demonstrate stability of the knee. An x-ray of the left knee shows moderate degenerative changes with medial joint compartment narrowing, small osteophytes, and a small effusion. The most likely cause for this patient’s symptoms is

  A. an anterior cruciate ligament tear
  B. gout
  C. iliotibial band syndrome
  D. a medial collateral ligament tear
  E. a medial meniscus tear
Explanation:

The correct answer is E. The two most common causes of meniscal tears are due to traumatic injury (often seen in athletes) and degenerative processes (seen in elderly patients who have more brittle cartilage). Individuals who experience a meniscal tear usually experience pain and swelling as the primary symptoms. Another common complaint is joint “locking”, or the inability to completely straighten the joint. This is due to a piece of the torn meniscus physically impinging the joint mechanism of the knee. An x-ray of the knee may either be normal or it may demonstrate degenerative changes in the knee joint.

An anterior cruciate ligament tear (choice A) is not likely the cause of this patient’s symptoms. The ACL is most commonly injured during sporting activities when an athlete suddenly pivots causing excessive rotational forces on the ligament. Other mechanisms that can cause an ACL tear include severe trauma and work injuries. Individuals who experience ACL tears usually describe a feeling of the joint “giving out” and people often report hearing a “pop”. An anterior drawer test will elicit instability of the knee joint such that the tibia is able to slide abnormally forward relative to the femur.

Gout (choice B) is not likely the cause of this patient’s symptoms. Gout is an inflammatory arthropathy that causes pain, swelling, and erythema in the affected joint. The sensation of the knee “locking” indicates some type of internal derangement of the knee as opposed to an inflammatory process.

Iliotibial band syndrome (choice C) is not likely the cause of this patient’s symptoms. This syndrome manifests as pain along the lateral aspect of the knee and is often seen in runners. The iliotibial band represents the tensor fascia lata muscle, which runs along the lateral aspect of the thigh and attaches to the lateral aspect of the tibia. The ligaments of the iliotibial band contribute fibers to the lateral collateral ligament. Lateral knee pain is due to irritation and inflammation of the distal portion of the iliotibial band at, or just distal to, the point at which it crosses the lateral femoral epicondyle. It is a type of overuse injury caused by repetitive friction of the iliotibial band across the lateral femoral epicondyle.

A medial collateral ligament tear (choice D) is not likely the cause of this patient’s symptoms. The MCL is rarely injured in isolation. It usually is part of the “unhappy triad”, used to describe injuries to the medial meniscus, anterior cruciate ligament, and the medial collateral ligament. This triad is seen in sports such as football, when the player is hit on the outside of the knee.

 

 

A 56-year-old woman comes to the office complaining of pain in the fingers of both of her hands. The pain is accompanied by stiffness, is worse in the mornings, and gradually improves throughout the day. Over the past few months she has also noticed some “lumps” on her forearms. Her past medical history is otherwise remarkable for mild hypertension and recent menopause. Her medications are only estrogen/progesterone for hormone replacement. Physical examination shows tenderness of the wrists and proximal joints of the second and third fingers, bilaterally. There are small, mobile nodules over the extensor surfaces of her forearms and hands. Laboratory studies will most likely show

  A. decreased serum calcium levels
  B. an elevated erythrocyte sedimentation rate (ESR)
  C. an elevated mean corpuscular volume
  D. hypercholesterolemia
  E. polycythemia
Explanation:

The correct answer is B. The patient has rheumatoid arthritis. She has all of the classical findings, which include morning stiffness, polyarticular involvement, and rheumatoid nodules. The etiology of RA is unknown but it is a progressive, destructive disease. The disease is seen in women at least two times more than men. It has a prevalence of nearly 10% worldwide in persons over the age of 65. Although most patients are rheumatoid factor positive, their levels do not correlate with the clinical severity of the disease. More consistently, ESR is elevated during active periods of the disease as well as during periods of remission.

Decreased serum calcium levels (choice A) are not characteristic of RA. Although patients at this age have issues concerning bone density, there is no reflection on the serum calcium levels. Total calcium levels may be abnormal however, but serum calcium is often unchanged.

Because anemia of chronic disease is a normocytic or microcytic anemia, the MCV (choice C) will be reduced, not elevated.

Hypercholesterolemia (choice D) is not characteristic of RA, but is a common clinical finding in nephritic syndrome. This patient’s age and postmenopausal status make her more likely to have high cholesterol compared to a younger, menstruating woman, but this is not necessarily the case.

Most patients with moderate RA suffer from anemia of chronic disease so they will not be polycythemic (choice E).

 

 

A young mother brings in her 3-year-old daughter to the clinic because of worsening acne problems on her daughter’s face. She had a normal birth without complications. However, she is developmentally slow and has had a few episodes of seizures of unknown etiology. Her medications include phenytoin and a multivitamin. The child is playful and her size and weight are appropriate for her age. She has numerous firm, flesh-colored papules scatter over her nose, both cheeks, and chin. There are no pustules or inflammatory papules. She also has multiple, small, hypopigmented ash leaf and confetti-type macules on both lower extremities. The rest of her examination is unremarkable. The next most appropriate step is to

  A. check morning level of cortisol
  B. check serum level of DHEAS and testosterone
  C. prescribe oral minocycline
  D. prescribe topical benzoyl peroxide every morning and topical erythromycin every evening
  E. send her for an x-ray of the skull
Explanation:

The correct answer is E. An x-ray of the skull is the most appropriate next step, because this patient has tuberous sclerosis (TS). TS is also known as epiloia (epi = epilepsy, loi = low intelligence, a = adenoma sebaceum). This is an autosomal dominant syndrome marked by other features including periungual fibromas, shagreen plaques, oral papillomatosis, ash leaf hypomelanotic macules, skin fibromas, and café au lait spots. Adenoma sebaceum are 1-3 mm, yellowish, red discrete, waxy papules that are distributed symmetrically over the cheeks, nose, and forehead. Histologically, these are angiofibromas. Congenital hypomelanotic macules are found in 85% of TS patients which may be detected at birth in most patients. They may be shaped like an ash leaf, but linear and confetti-type white macules may be present. The hypomelanotic macules are an indication for a skull x-ray evaluation to search for calcified intracranial nodules. These are hamartomatous proliferations of glial and neuronal tissue produce potato-like nodules or brain stones in the cortex leading to seizures.

Checking levels of cortisol (choice A) and DHEAS/ testosterone (choice B) is incorrect because this patient does not show other signs of endocrine abnormalities (i.e., hypertrichosis and enlarged external genitalia).

Prescribing minocycline (choice C) and benzoyl peroxide and erythromycin (choice D) is incorrect, because these treatments are aimed at treating acne lesions.

 

 

A 51-year old woman with systemic lupus erythematosus comes to the clinic with right hip pain for the past 3 days. She has no history of trauma. The patient has been postmenopausal for the past 3 years. She takes naproxen, ibuprofen, and prednisone 100 mg a day. She has been taking these medications for the past 10 years. She tells you that she stopped taking estrogen secondary to occasional nausea. Her temperature is 37.4 C (99.4 F). Physical examination reveals a butterfly facial rash, multiple swollen joints of the hands and feet, and a systolic flow murmur. A plain radiograph of the hip is normal. A bone scan reveals decreased 99Tc MDP tracer uptake in the right femoral head. The patient should be advised to

  A. avoid weightbearing and immediately stop prednisone
  B. avoid weightbearing and taper off prednisone
  C. begin an exercise program and calcium supplementation
  D. begin an exercise program and taper off prednisone
  E. begin an exercise program, calcium supplementation, and estrogen replacement therapy
Explanation:

The correct answer is B. The history, physical exam, and bone scan results are consistent with systemic lupus erythematosus and avascular necrosis (AVN) of the right femoral head. This is a common complication of long-term steroid therapy. AVN may also be a result of pancreatitis, fat embolus, alcoholism, sickle cell anemia, air emboli, or idiopathic causes as in Legg-Calve-Perthes disease. Avoidance of activity, especially weightbearing is necessary to ensure that the femoral head does not collapse and fracture. Given a prednisone dose greater than 10mg QD, a slow taper should be initiated to allow the adrenal glands to resume normal cortisol production.

A rapid taper of prednisone (choice A) could result in adrenal insufficiency. Symptoms of this iatrogenic condition include fatigue, tiredness, depression, and weight loss. More severe cases result in hypotension, irregular pulse, and nausea.

Exercise involving the lower extremities (choice C) must be avoided as collapse and fracture of the femoral head may occur. AVN is not a symptom of osteoporosis, so standard osteoporosis treatments do not apply.

Exercise involving the lower extremities (choice D) must be avoided as collapse and fracture of the femoral head may occur.

Exercise involving the lower extremities (choice E) must be avoided as collapse and fracture of the femoral head may occur. AVN is not a symptom of osteoporosis, so standard osteoporosis treatments do not apply. Since this patient is postmenopausal, estrogen replacement therapy should be begun as a separate issue.

 

 

A 44-year-old Asian man comes to the office for a health maintenance examination. He does not take any medications routinely and has no diagnosed medical problems. He does not smoke or drink alcohol on regular basis. On several occasions, he has tried diet and exercise to shed some weight without success. He is an obese male with hyperhidrosis. There is hyperpigmentation with a velvety appearance on the nape and bilateral axillae. He has similar lesions in the groin area. Oral mucosa and palmoplantar surfaces are unremarkable. The abnormal laboratory test result that is most likely correlated with these findings is an elevated

  A. chorionic embryo antigen
  B. hemoglobin A1C
  C. testosterone
  D. total cholesterol
  E. triglyceride
Explanation:

The correct answer is B. Acanthosis nigricans (AN) is characterized by hyperpigmentation and papillary hypertrophy, which are symmetrically distributed. Different types of AN have been described. The most common variety of AN is grayish, velvety thickening of the skin of the sides of the neck, axillae, and groin. It occurs in obesity, insulin-resistance, with or without endocrine disorders such as diabetes mellitus and hypothyroidism as well as Cushing’s syndrome and Addison’s disease. Therefore, an elevated hemoglobin A1C, which is associated with diabetes, is correlated with these findings.

High chorionic embryo antigen (choice A) is incorrect. One of the rare variants of AN is the malignant type which may either precede, accompany, or follow the onset of internal cancer. Most cases are associated with adenocarcinoma, especially of the gastrointestinal tract, lung and breast. Less often, the gallbladder, pancreas, esophagus, liver, prostate, kidney, colon, rectum, uterus, and ovaries are involved.

High testosterone level (choice C) is incorrect. Sex hormones has not been associated with AN.

High total cholesterol (choice D) and high triglyceride (choice E) are both incorrect. Although, these two findings are likely in an obese patient, they are not directly related to AN.

 

 

A 35-year-old elementary school art teacher comes into the office worried about a growing lesion on her left thumb over the last 3-4 weeks. She recalls working with sharp knives and accidentally stabbing the thumb in sculpture class around the same time she noticed the lesion. She tells you that initially the lesion looked liked a small “pimple” which grew in size, bleeding when irritated, and has rough surfaces. She describes a “pulsing” tender sensation. Physical examination shows a 1.2 cm x 1.0 cm polypoid, soft, red mass constricted by a collarette at the base of the lesion located on the lateral nail fold of the left thumb. No pulse over the lesion is appreciated on palpation. The remainder of the cutaneous examination is unremarkable. The most appropriate next step is to

  A. check HIV status because this is most likely Kaposi sarcoma
  B. shave and electrodesiccate the base of the lesion and send the specimen to pathology for analysis
  C. shave the lesion off and throw it away because it is a benign lesion
  D. tell her this is an acquired hemangioma and it will involute on its own
  E. tell her this is a fungal infection and you need to treat it with oral antifungal agents
Explanation:

The correct answer is B. A pyogenic granuloma is a result of exuberant granulation tissue following minor trauma (usually with sharp objects) or infection of the skin. It presents as an intensely red, sometimes ulcerated, pedunculated mass that bleeds easily on provocation. Treatment usually involves removal of the mass by either shave and electrodessication or complete excision. Clinically, it can resemble amelanotic melanoma; therefore, always submit the specimen for histologic confirmation.

Kaposi sarcoma (KS) (choice A) is incorrect because HIV-associated lesions of KS are generally asymptomatic and frequently seen on the soles of feet, the hard palate, and the tip of the nose. Other common sites include face, trunk, and penis. In this setting it occurs predominantly among homosexual men. It is associated with an increased number of sexual partners, a prior history of oropharyngeal gonorrhea, and increased contact with a sex partner’s feces through oral-anal contact.

Shave the lesion off and throw it away because it is a benign lesion (choice C) is incorrect because you should never shave a lesion without evaluating it for histologic confirmation. There have been cases reported of amelanotic melanomas that were removed with a strong clinical suspicion of pyogenic granuloma.

Acquired hemangiomas, also known as cherry angiomas, (choice D) is incorrect because these lesions are small, bright-red, papules composed of ecstatic vessels. They generally arise in middle age, usually on the abdomen. They tend not to grow rapidly in size and do not involute spontaneously.

Fungal infection (choice E) is incorrect because infection of the nails tend to result in dystrophic, thickened, discolored nail plates with generalized erythema and scaling of the proximal nail fold.

 

 

A 66-year-old woman comes to the clinic because of a 3-month history of increasing pain in her right knee that began after a very long car trip across the country. She has a severe, sharp, non-radiating pain primarily in the medial, but also extending to the inferior aspect of her right knee. It occurs anytime of the day or night and sometimes awakens her from sleep. There is no activity that decreases the pain or initiates it. She says she also “loses feeling” in her right lateral thigh sometimes and there is often a “throbbing feeling” in different parts of her right leg. She denies urinary problems or symptoms in the left leg. She takes ibuprofen for the pain even though it is “not very effective.” Physical therapy and deep massage have “not worked and have even made things worse.” She had right leg pain 15 years earlier, which was treated with a lumbar laminectomy and fusion. She does not remember exactly what the original pain felt like, but since the surgery it has been much better. There was an episode of increased pain 5 years ago, but this resolved on its own. She had seen a different physician 5 weeks ago for this knee pain who sent her for an MRI of the lumbar spine, which showed L4-5 stable appearing spondylolisthesis and no significant root or cord pathology. She also had bilateral knee x-rays at that time, which showed mild to moderate arthritic changes. Physical examination shows a normal motor exam, loss of pinprick sensation over most of the right lateral thigh, tenderness to palpation of the right femoral nerve and to deep palpation of the medial knee. Her reflexes are all normal and symmetric. The most appropriate next step in the management of this patient is to

  A. give her a prescription for morphine
  B. order an MRI of the thoracic spine
  C. refer her for neurosurgical evaluation
  D. refer her for orthopedic evaluation
  E. repeat the lumbar spine MRI
Explanation:

The correct answer is D. This patient presents with localized knee pain, which is exacerbated by palpation of the medial knee. It is most likely due to structural knee pathology, perhaps of the medial meniscus. There are no elements of the history or exam that suggest that the pain is radicular. The right lateral thigh symptoms are probably due to femoral nerve entrapment and warrant no clinical intervention if they are not bothersome.

Merely treating the pain without a proper evaluation and diagnosis would be a mistake (choice A). Structural knee pathology is often treatable surgically.

An MRI of the thoracic spine (choice B) or lumbar spine (choice E) are not indicated as there is no evidence of spine or root pathology.

Since there is no reason to suspect a new neurosurgical problem based on the above explanation, neurosurgical consult is not indicated (choice C).